You are on page 1of 119

Quantity A: (-6)4

Quantity B: (-6)5

A. if the quantity A is greater;


B. if the quantity B is greater;
C. if the two quantities are equal;
D. if the relationship cannot be determined from the information given.

Ans : A

Quantity A: Time to travel 95 miles at 50 miles per hour


Quantity B: Time to travel 125 miles at 60 miles per hour

A. Quantity A is greater
B. Quantity A equals Quantity B
C. Quantity B is greater
D. Relationship Indeterminate

Ans : C

Quantity A: (9/13)2
Quantity B: (9/13)1/2

A. Quantity A equals Quantity B


B. Relationship Indeterminate
C. Quantity B is greater
D. Quantity A is greater

Ans : C

Quantity A: 4 / 100
Quantity B: 0.012 / 3

A. Quantity B is greater
B. Quantity A equals Quantity B
C. Quantity A is greater
D. Relationship Indeterminate

Ans : C

x = 2y + 3
y = -2

Quantity A: x
Quantity B: -1

A. if the quantity in Column A is greater


B. if the quantity in Column B is greater
C. if the two quantities are equal
D. if the relationship cannot be determined from the information given
Ans : C

x + 2y > 8

Quantity A: 2x + 4y
Quantity B: 20

A. if the quantity in Column A is greater


B. if the quantity in Column B is greater
C. if the two quantities are equal
D. if the relationship cannot be determined from the information given.

Ans : D

Quantity A: The number of months in 7 years


Quantity B: The number of days in 12 weeks

A. if the quantity in Column A is greater


B. if the quantity in Column B is greater
C. if the two quantities are equal
D. if the relationship cannot be determined from the information given

Ans : C

Quantity A: 1-1/27
Quantity B: 8/9 + 1/81

A. if the quantity in is greater


B. if the quantity in is greater
C. if the two quantities are equal
D. if the relationship cannot be determined from the information given.

Ans : A

r/>s/>0/>

Quantity A: rs/r
Quantity B: rs/s

A. if the quantity A is greater


B. if the quantity B is greater
C. if the two quantities are equal
D. if the relationship cannot be determined from the information given.

Ans : B

Quantity A: 0.83
Quantity B: 0.81/3

A. Quantity B is greater
B. Relationship Indeterminate
C. Quantity A is greater
D. Quantity A equals Quantity B

Ans : A

11. Ans : D
12. Quantity A: (0.82)2(0.82)3
Quantity B:(0.82)6
A. if the quantity in Column A is greater;
B. if the quantity in Column B is greater;
C. if the two quantities are equal;
D. if the relationship cannot be determined from the information given.

Ans : A

13. For all real numbers a, let a* = 1 - a.

Quantity A: ((-1)*)*
Quantity B: 2*

A. if the quantity in Column A is greater;


B. if the quantity in Column B is greater;
C. if the two quantities are equal;
D. if the relationship cannot be determined from the information given.

Ans : C

14. Quantity A: (x - 1)(x)(x + 1)


Quantity B:(x)(x)(x)
A. if the quantity in Column A is greater;
B. if the quantity in Column B is greater;
C. if the two quantities are equal;
D. if the relationship cannot be determined from the information given.

Ans : D

15. Quantity A: (3 x 4 x 17) / (121 x 100)


Quantity B: (4 x 5 x 19) / (1000 x 121)
A. Quantity A is greater
B. Quantity A equals Quantity B
C. Relationship Indeterminate
D. Quantity B is greater

16. Consider a triangle PQR.

Quantity A: length of PQ + length of QR


Quantity B: length of PR
A. Quantity A is greater
B. Quantity B is greater
C. Relationship Indeterminate
D. Quantity A equals Quantity B

Ans : A

17. Quantity A: (27 - 13) (296 + 534)


Quantity B: (27 + 13) (534 + 296)
A. Quantity B is greater
B. Quantity A equals Quantity B
C. Relationship Indeterminate
D. Quantity A is greater

Ans : D

18. Quantity A: A = 1.1


Quantity B: 12.11/2
A. Relationship Indeterminate
B. Quantity B is greater
C. Quantity A equals Quantity B
D. Quantity A is greater

Ans : B

19. 100 < y < 200 and 100 < z < 210

Quantity A: y
Quantity B: z

A. Quantity A is greater
B. Quantity A equals Quantity B
C. Quantity B is greater
D. Relationship Indeterminate

Ans : D

20. y2 + z2 = 34 and yz = 15

Quantity A: y2 + 2yz + z2
Quantity B: (y + z)2

A. Quantity B is greater
B. Relationship Indeterminate
C. Quantity A is greater
D. Quantity A equals Quantity B

Ans : D
Quantitative Comparisons
21. Consider a rectangle. The length of its shorter side is 8, and the length of its
diagonal is 16.

30o
Quantity A:
Quantity B: measure of angle formed by diagonal and shorter side

A. Relationship Indeterminate
B. Quantity A equals Quantity B
C. Quantity A is greater
D. Quantity B is greater

Ans : D

22. Quantity A: (y + 5)2


Quantity B: (y - 5)2
A. Quantity B is greater
B. Relationship Indeterminate
C. Quantity A equals Quantity B
D. Quantity A is greater

Ans : B

23. Quantity A: (1/25)1/2 + (1/144)1/2


Quantity B: [(1/25) + (1/144)]1/2
A. Relationship Indeterminate
B. Quantity A is greater
C. Quantity B is greater
D. Quantity A equals Quantity B

Ans : A

24. y2 + z2 = 34 and yz = 15

Quantity A: y2 + 2yz + z2
Quantity B: (y + z)2

A. Quantity A is greater
B. Relationship Indeterminate
C. Quantity A equals Quantity B
D. Quantity B is greater

Ans : C

25. 100 < y < 200 and 100 < z < 210

Quantity A: y
Quantity B: z
A. Quantity A is greater
B. Quantity A equals Quantity B
C. Quantity B is greater
D. Relationship Indeterminate

Ans : D

26. Quantity A: (y + 5)2


Quantity B: (y - 5)2
A. Quantity A equals Quantity B
B. Quantity A is greater
C. Relationship Indeterminate
D. Quantity B is greater

Ans : C

27. Consider a rectangle. The length of its shorter side is 8, and the length of its
diagonal is 16.

Quantity A: 30o
Quantity B: measure of angle formed by diagonal and shorter side

A. Quantity A is greater
B. Quantity A equals Quantity B
C. Quantity B is greater
D. Relationship Indeterminate

Ans : C

28. The sum of three consecutive even numbers is 18.

Quantity A: Their average


Quantity B: 6

A. Relationship Indeterminate
B. Quantity A is greater
C. Quantity A equals Quantity B
D. Quantity B is greater

Ans : C

29. x - y > 10

Quantity A: y - x
Quantity B: 12

A. Quantity B is greater
B. Quantity A is greater
C. Quantity A equals Quantity B
D. Relationship Indeterminate
Ans : A

30. x = 0, y > 0

Quantity A: xy
Quantity B: yx

A. Quantity A equals Quantity B


B. Quantity A is greater
C. Quantity B is greater
D. Relationship Indeterminate

Ans : C

Quantitative Comparisons

31.

Diagram is illustrative and is not drawn to scale.

Quantity A: Measure
of angle 3 - Measure of angle 2
Quantity B: Measure of angle 5 - Measure of angle 6

A. Relationship Indeterminate
B. Quantity A equals Quantity B
C. Quantity B is greater
D. Quantity A is greater

Ans : B

32. Quantity A: 29
Quantity B: 92
A. Quantity B is greater
B. Quantity A is greater
C. Relationship Indeterminate
D. Quantity A equals Quantity B

Ans : B
33. 0 < -x < 10
11 < -y < 20

Quantity A: x
Quantity B: y

A. Relationship Indeterminate
B. Quantity A equals Quantity B
C. Quantity B is greater
D. Quantity A is greater

Ans : D

34.

Diagram is illustrative and is not drawn to scale.


Given angles 1 and 2 are equal,
length of side AB = x, length of side BC = y, length of side AC = z.

Quantity A: x + y
Quantity B: y + z

A. Quantity A is greater
B. Quantity A equals Quantity B
C. Quantity B is greater
D. Relationship Indeterminate

Ans : B

35.

Diagram is illustrative and is not drawn to scale.


In triangle ABC, AB = AC and measure of angle 1 = 100o.
Quantity A: Measure of angle 2 + Measure of angle 3
Quantity B: 90o

A. Quantity B is greater
B. Relationship Indeterminate
C. Quantity A is greater
D. Quantity A equals Quantity B

Ans : A

36. x and y are both positive and x/y > 5

Quantity A: 0.2x
Quantity B: y

A. Quantity A is greater
B. Quantity B is greater
C. Relationship Indeterminate
D. Quantity A equals Quantity B

Ans : A

37.

Diagram is illustrative and is not drawn to scale.


Given AB = AC and angle BAC = 60o

Quantity A: Length of side AB


Quantity B: Length of side BC

A. Quantity A equals Quantity B


B. Quantity B is greater
C. Relationship Indeterminate
D. Quantity A is greater

Ans : A

38. y2 = 36

Quantity A: y
Quantity B: 6
A. Relationship Indeterminate
B. Quantity A is greater
C. Quantity B is greater
D. Quantity A equals Quantity B

Ans : A

39.

Diagram is illustrative and is not drawn to scale.

Quantity A: Measure of angle 1 + Measure of angle 2 + Measure of angle 4


Quantity B: 180o

A. Relationship Indeterminate
B. Quantity A is greater
C. Quantity B is greater
D. Quantity A equals Quantity B

Ans : D

40.

Diagram is illustrative and is not drawn to scale.


In triangle ABC, angle A = 60o and AB = AC.

Quantity A: Measure of angle 1 + Measure of angle 2


Quantity B: 120o

A. Relationship Indeterminate
B. Quantity A is greater
C. Quantity A equals Quantity B
D. Quantity B is greater

Ans : C
Quantitative Comparisons

41.

Diagram is illustrative and is not drawn to scale.

Quantity A: Measure of angle 2 + Measure of angle 3


o
Quantity B: 180

A. Quantity B is greater
B. Quantity A is greater
C. Quantity A equals Quantity B
D. Relationship Indeterminate

Ans : C

42.

Diagram is illustrative and is not drawn to scale.


AB is the diameter of the circle.

Quantity A: Measure of angle 1


Quantity B: Measure of angle 2

A. Relationship Indeterminate
B. Quantity A is greater
C. Quantity B is greater
D. Quantity A equals Quantity B

Ans : D
43.

Diagram is illustrative and is not drawn to scale.

Quantity A: Measure of angle 1 + Measure of angle 3


Quantity B: Measure of angle 2 + Measure of angle 4

A. Relationship Indeterminate
B. Quantity A is greater
C. Quantity A equals Quantity B
D. Quantity B is greater

Ans : A

44.

Diagram is illustrative and is not drawn to scale.


In triangle ABC, AB = AC and measure of angle 1 = 100o.

Quantity A: Measure of angle 2 + Measure of angle 3


Quantity B: 90o

A. Quantity B is greater
B. Quantity A equals Quantity B
C. Relationship Indeterminate
D. Quantity A is greater

Ans : A
45.

Diagram is illustrative and is not drawn to scale.


Given angles 1 and 2 are equal,
length of side AB = x, length of side BC = y, length of side AC = z.

Quantity A: x + y
Quantity B: y + z

A. Quantity B is greater
B. Quantity A equals Quantity B
C. Quantity A is greater
D. Relationship Indeterminate

Ans : B

46. x and y are both positive and x/y > 5

Quantity A: 0.2x
Quantity B: y

A. Quantity B is greater
B. Relationship Indeterminate
C. Quantity A equals Quantity B
D. Quantity A is greater

Ans : D

47. yz < 0

Quantity A: (y - z)2
Quantity B: y2 + z2

A. Quantity A is greater
B. Quantity B is greater
C. Quantity A equals Quantity B
D. Relationship Indeterminate

Ans : A

48. For any positive integer n,


n! is the product of all positive integers less than or equal to n.
Quantity A: 20! / 17!
Quantity B: 80! / 78!

A. Quantity A is greater
B. Quantity B is greater
C. Quantity A equals Quantity B
D. Relationship Indeterminate

Ans : A

49. 2 < z < 4

Quantity A: π2z3
Quantity B: π3z2

A. Quantity A is greater
B. Quantity B is greater
C. Quantity A equals Quantity B
D. Relationship Indeterminate

Ans : D

50. Amy, Beth and Charlie divided a pizza amongst themselves.


Amy took 30% of the pizza and ate (3/4) of what she took.
Beth took 20% of the pizza.
Charlie ate (2/5) of what he took.

Quantity A: The amount Amy ate


Quantity B: The amount Charlie ate

A. Quantity A is greater
B. Quantity B is greater
C. Quantity A equals Quantity B
D. Relationship Indeterminate

Ans : A

ative Comparisons
51. p > 0 > q

Quantity A:
p+q
Quantity B: pq

A. The quantity in Column A is greater.


B. The quantity in Column B is greater.
C. The quantities are equal.
D. The relationship cannot be determined from the information given.

Ans : D

52. Quantity A: The average (arithmetic mean) of x and y


Quantity B: The average (arithmetic mean) of x - 1 and y + 1
A. The quantity in Column A is greater.
B. The quantity in Column B is greater.
C. The quantities are equal.
D. The relationship cannot be determined from the information given.

Ans : C

53. The integer (x - 1) is a prime number between 40 and 50.

Quantity A: The sum of all different prime factors of x


Quantity B: 14

A. The quantity in Column A is greater.


B. The quantity in Column B is greater.
C. The quantities are equal.
D. The relationship cannot be determined from the information given.

Ans : B

54. A < C
B>D>0

Quantity A: A - B
Quantity B: C - D

A. The quantity in Column A is greater.


B. The quantity in Column B is greater.
C. The quantities are equal.
D. The relationship cannot be determined from the information given.

Ans : B

55. In a particular jellybean jar, the number of red jellybeans exceeds the number of
white ones by a ratio of 3:2. If two red jellybeans were removed, the ratio of red to
white jellybeans would be 1:1.

Quantity A: The number of white jellybeans in the jar


Quantity B: 4

A. The quantity in Column A is greater.


B. The quantity in Column B is greater.
C. The quantities are equal. be determined from the information given.

Ans : C
D. tionship cannot

The relauantitative Section : Quantitative


Ability
Directions: Solve each CAT sample quantitative ability problem and indicate the best of
the answer choices given..

Numbers: All numbers used are real numbers.

Figures: A figure accompanying a CAT sample quantitatove ability problem solving


question is intended to provide information useful in solving the problem. Figures are
drawn as accurately as possible EXCEPT when it is stated in a specific problem that its
figure is not drawn to scale. Straight lines may sometimes appear jagged. All figures lie on
a plane unless otherwise indicated.

Following are some CAT sample quantitative ability questions.

1. A rectangle is 14 cm long and 10 cm wide. If the length is reduced by x cms and its
width is increased also by x cms so as to make it a square then its area changes by :
A. 4
B. 144
C. 12
D. 2
E. None of the above.

Ans : A

2. A motorcycle stunts man belonging to a fair, rides over the vertical walls of a
circular well at an average speed of 54 kph for 5 minutes. If the radius of the well is
5 meters then the distance traveled is:
A. 2.5 kms
B. 3.5 kms
C. 4.5 kms
D. 5.5 kms
E. None of the above

Ans : C

3. If 1 cm on a map corresponds to an actual distance of 40 kms. And the distance on


the map between Bombay and Calcutta is 37.5 cms., the actual distance between
them is :
A. 375 kms
B. 3750 kms
C. 1500 kms
D. 1375 kms
E. None of the above
Ans : C

4. A box contains 90 mts each of 100 gms and 100 bolts each of 150 gms. If the entire
box weighs 35.5 kg., then the weight of the empty box is :
A. 10 kg
B. 10.5 kg
C. 11 kg
D. 11.5 kg
E. None of the above

Ans : D

5. If the radius of a circle is increased by 20% then the area is increased by :


A. 44%
B. 120%
C. 144%
D. 40%
E. None of the above

Ans : A

6. Tom, Dick and Harry went for lunch to a restaurant. Tom had $100 with him, Dick
had $60 and Harry had $409. They got a bill for $104 and decided to give a tip of
$16. They further decided to share the total expenses in the ratio of the amounts of
money each carried. The amount of money which Tom paid more than what Harry
paid is
A. 120
B. 200
C. 60
D. 24
E. 36

Ans : E

7. A plot of land is in the shape of a trapezium whose dimensions are given in the figure
below :

Hence the perimeter of the field is


A. 50 m
B. 64 m
C. 72 m
D. 84 m
E. None of the above

Ans : c
8. Four concentric ( having the same center ) circles with radii, x, 2x, 3x and 4x are
drawn to form two rings A and B as shown in the figure.

Ratio of the area of inner ring A to the area of outer ring B is


A. 1 : 2
B. 1 : 4
C. 2 : 3
D. 3 : 7
E. None of the above

Ans : D

9. If 3/p = 6 and 3/q = 15 then p - q = ?


A. 1/3
B. 2/5
C. 3/10
D. 5/6
E. None of the above

Ans : C

10. A father is three times as old as his son. After fifteen years the father will be twice
as old as his son's age at that time. Hence the father's present age is
A. 36
B. 42
C. 45
D. 48
E. None of the above

Ans : C

titative Section : Quantitative Ability


11. (1/4)3 + (3/4)3 + 3(1/4)(3/4)(1/4 + 3/4) =?
A. 1/64
B. 27/64
C. 49/64
D. 0
E. 1

Ans : E
12. If the area of two circles are in the ratio 169 : 196 then the ratio of their radii is
A. 10 : 11
B. 11 : 12
C. 12 : 13
D. 13 : 14
E. None of the above

Ans : D

13. A semi-circle is surmounted on the side of a square. The ratio of the area of the
semi-circle to the area of the square is

A. 1:2
B. 2:p
C. p:8
D. 8:p
E. None of the above

Ans : C

14. Which of the following is the greatest ?


A. 40% of 30
B. 3/5 of 25
C. 6.5% of 200
D. Five more than the square of 3
E. 1/2-4

Ans : E

15. Two identical taps fill 2/5 of a tank in 20 minutes. When one of the taps goes dry in
how many minutes will the remaining one tap fill the rest of the tank ?
A. 5 minutes
B. 10 minutes
C. 15 minutes
D. 20 minutes
E. None of the above

Ans : C

16. If the value of XYZ Company stock drops from $25 per share to $21 per share,
what is the percent of the decrease?
A. 4
B. 8
C. 12
D. 16
E. 20

Ans : D

17. If a building b feet high casts a shadow f feet long, then, at the same time of day, a
tree t feet high will cast a shadow how many feet long?
A. ft/b
B. fb/t
C. b/ft
D. tb/f
E. t/fb

Ans : A

18. If x, y, and z are consecutive negative integers, and if x > y > z, which of the
following must be a positive odd integer?
A. xyz
B. (x - y) (y - z)
C. x - yz
D. x(y + z)
E. x + y + z

Ans : B

19. At a certain ice cream parlor, customers can choose among five different ice cream flavors
and can choose either a sugar cone or a waffle cone. Considering both ice cream
flavor and cone type, how many distinct triple-scoop cones with three different ice
cream flavors are available?
A. 12
B. 16
C. 20
D. 24
E. 30

Ans : C

20. What is the greatest value of a positive integer n such that 3n is a factor of 1815?
A. 15
B. 18
C. 30
D. 33
E. 45

Ans : C

Quantitative Section : Quantitative Ability


21. If .2t = 2.2 - .6s and .5s = .2t + 1.1, then s =
A. 1
B. 3
C. 10
D. 11
E. 30

Ans : B

22. Five years ago, Beth's age was three times that of Amy. Ten years ago, Beth's age was
one half that of Chelsea. If C repre- sents Chelsea's current age, which of the
following represents Amy's current age?
A. c/6 + 5
B. 2c
C. (c-10)/3
D. 3c-5
E. 5c/3 - 10

Ans : A

23. A portion of $7200 is invested at a 4% annual return, while the remainder is


invested at a 5% annual return. If the annual income from both portions is the same,
what is the total income from the two investments?
A. $160
B. $320
C. $400
D. $720
E. $1,600

Ans : B

24. An empty swimming pool can be filled to capacity through an inlet pipe in 3 hours,
and it can be completely drained by a drainpipe in 6 hours. If both pipes are fully
open at the same time, in how many hours will the empty pool be filled to capacity?
A. 4
B. 4.5
C. 5
D. 5.5
E. 6

Ans : E

25. If r = (3p + q)/2 and s = p - q, for which of the following values of p would r2 = s2?
A. 1q/5
B. 10 - 3q/2
C. q - 1
D. 3q
E. 9q/2 - 9

Ans : A
26. At 10 a.m. two trains started traveling toward each other from stations 287 miles
apart. They passed each other at 1:30 p.m. the same day. If the average speed of the
faster train exceeded the average speed of the slower train by 6 miles per hour,
which of the following represents the speed of the faster train, in miles per hour?
A. 38
B. 40
C. 44
D. 48
E. 50

Ans : C

27. On the xy-coordinate plane, points A and B both lie on the circumference of a circle
whose center is O, and the length of AB equals the circle's diameter. If the (x,y)
coordinates of O are (2,1) and the (x,y) coordinates of B are (4,6), what are the
(x,y) coordinates of A?
A. (3, 3/2)
B. (1, 2/2)
C. (0, -4)
D. (2/2, 1)
E. (-1, -2/2)

Ans : C

28. If a rectangle's length and width are both doubled, by what percent is the rectangle's
area increased?
A. 50
B. 100
C. 200
D. 300
E. 400

Ans : D

29. A rectangular tank 10" by 8" by 4" is filled with water. If all of the water is to be
transferred to cube-shaped tanks, each one 3 inches on a side, how many of these
smaller tanks are needed?
A. 9
B. 12
C. 16
D. 21
E. 39

Ans : B

30. Point Q lies at the center of the square base (ABCD) of the pyramid pictured above.
The pyramid's height (PQ) measures exactly one half the length of each edge of its
base, and point E lies exactly halfway between C and D along one edge of the base.
What is the ratio of the surface area of any of the pyramid's four triangular faces to
the surface area of the shaded triangle?
A. 3 :√2
B. √5:1
C. 4√3:3
D. 2√2:1
E. 8:√5

Ans : D

Quantitative Section : Quantitative Ability


Average

31. The average wages of a worker during a fortnight comprising 15 consecutive


working days was Rs.90 per day. During the first 7 days, his average wages was
Rs.87/day and the average wages during the last 7 days was Rs.92 /day. What was
his wage on the 8th day?
A. 83
B. 92
C. 90
D. 97

Ans : D

32. The average of 5 quantities is 6. The average of 3 of them is 8. What is the average
of the remaining two numbers?
A. 6.5
B. 4
C. 3
D. 3.5

Ans : C

33. The average temperature on Wednesday, Thursday and Friday was 250. The
average temperature on Thursday, Friday and Saturday was 240. If the temperature
on Saturday was 270, what was the temperature on Wednesday?
A. 240
B. 210
C. 270
D. 300

Ans : D

34. The average age of a group of 12 students is 20years. If 4 more students join the
group, the average age increases by 1 year. The average age of the new students is
A. 24
B. 26
C. 23
D. 22
Ans : A

35. When a student weighing 45 kgs left a class, the average weight of the remaining 59
students increased by 200g. What is the average weight of the remaining 59
students?
A. 57 kgs
B. 56.8 kgs
C. 58.2 kgs
D. 52.2 kgs

Ans : A

36. Three math classes: X, Y, and Z, take an algebra test.


The average score in class X is 83.
The average score in class Y is 76.
The average score in class Z is 85.
The average score of all students in classes X and Y together is 79.
The average score of all students in classes Y and Z together is 81.

What is the average for all the three classes?

A. 81
B. 81.5
C. 82
D. 84.5

Ans : B

37. The average weight of a class of 24 students is 36 years. When the weight of the
teacher is also included, the average weight increases by 1kg. What is the weight of
the teacher?
A. 60 kgs
B. 61 kgs
C. 37 kgs
D. None of these

Ans : B

38. The average of 5 quantities is 10 and the average of 3 of them is 9. What is the
average of the remaining 2?
A. 11
B. 12
C. 11.5
D. 12.5

Ans : C

39. The average age of a family of 5 members is 20 years. If the age of the youngest
member be 10 years then what was the average age of the family at the time of the
birth of the youngest member?
A. 13.5
B. 14
C. 15
D. 12.5

Ans : D

40. A student finds the average of 10 positive integers. Each integer contains two digits.
By mistake, the boy interchanges the digits of one number say ba for ab. Due to
this, the average becomes 1.8 less than the previous one. What was the difference
of the two digits a and b?
A. 8
B. 6
C. 2
D. 4

Ans : C

41. Average cost of 5 apples and 4 mangoes is Rs. 36. The average cost of 7 apples and 8
mangoes is Rs. 48. Find the total cost of 24 apples and 24 mangoes.
A. 1044
B. 2088
C. 720
D. 324

Ans : B

Quantitative Section : Quantitative Ability


Interest

41. A father left a will of Rs.35 lakhs between his two daughters aged 8.5 and 16 such
that they may get equal amounts when each of them reach the age of 21 years. The
original amount of Rs.35 lakhs has been instructed to be invested at 10% p.a. simple
interest. How much did the elder daughter get at the time of the will?
A. Rs. 17.5 lakhs
B. Rs. 21 lakhs
C. Rs. 15 lakhs
D. Rs. 20 lakhs

Ans : B

42. What will Rs.1500 amount to in three years if it is invested in 20% p.a. compound
interest, interest being compounded annually?
A. 2400
B. 2592
C. 2678
D. 2540
Ans : B

43. If a sum of money grows to 144/121 times when invested for two years in a scheme
where interest is compounded annually, how long will the same sum of money take
to treble if invested at the same rate of interest in a scheme where interest is
computed using simple interest method?
A. 9 years
B. 22 years
C. 18 years
D. 33 years

Ans : B

44. The population of a town was 3600 three years back. It is 4800 right now. What
will be the population three years down the line, if the rate of growth of population
has been constant over the years and has been compounding annually?
A. 6000
B. 6400
C. 7200
D. 9600

Ans : B

45. A man invests Rs.5000 for 3 years at 5% p.a. compound interest reckoned yearly.
Income tax at the rate of 20% on the interest earned is deducted at the end of each
year. Find the amount at the end of the third year.
A. 5624.32
B. 5630.50
C. 5788.125
D. 5627.20

Ans : A

46. The difference between the compound interest and the simple interest on a certain sum at
12% p.a. for two years is Rs.90. What will be the value of the amount at the end of
3 years?
A. 9000
B. 6250
C. 8530.80
D. 8780.80

Ans : D

47. Vijay invested Rs.50,000 partly at 10% and partly at 15%. His total income after a
year was Rs.7000. How much did he invest at the rate of 10%?
A. Rs.40,000
B. Rs.40,000
C. Rs.12,000
D. Rs.20,000
Ans : B

48. A sum of money invested for a certain number of years at 8% p.a. simple interest
grows to Rs.180. The same sum of money invested for the same number of years at
4% p.a. simple interest grows to Rs.120 only. For how many years was the sum
invested?
A. 25 years
B. 40 years
C. 33 years and 4 months
D. Cannot be determined

Ans : A

49. How long will it take for a sum of money to grow from Rs.1250 to Rs.10,000, if it
is invested at 12.5% p.a simple interest?
A. 8 years
B. 64 years
C. 72 years
D. 56 years

Ans : D

50. Rs. 5887 is divided between Shyam and Ram, such that Shyam's share at the end of
9 years is equal to Ram's share at the end of 11 years, compounded annually at the
rate of 5%. Find the share of Shyam.
A. 2088
B. 2000
C. 3087
D. None of these

Ans : C

Quantitative Section : Quantitative Ability


Geometry

51. Find the coordinates of the point which divides the line joining (5, -2) and (9, 6)
internally in the ratio 1 : 3.
A. (6, 0)
B. (6, 3)
C. (0, 6)
D. (3, 6)

Ans : A

52. Find the number of triangles in an octagon.


A. 326
B. 120
C. 56
D. Cannot be determined

Ans : C

53. Find the equation of a line whose intercepts are twice of the line 3x - 2y - 12 = 0
A. 3x - 2y = 24
B. 2x - 3y = 12
C. 2x - 3y = 24
D. None of these

Ans : A

54. Find the area of the sector covered by the hour hand after it has moved through 3
hours and the length of the hour hand is 7cm.
A. 77 sq.cm
B. 38.5 sq.cm
C. 35 sq.cm
D. 70 sq.cm

Ans : B

55. Find the area of the triangle whose vertices are (-6, -2), (-4, -6), (-2, 5).
A. 36
B. 18
C. 15
D. 30

Ans : C

56. A stairway 10ft high is such that each step accounts for half a foot upward and one-
foot forward. What distance will an ant travel if it starts from ground level to reach
the top of the stairway?
A. 30 ft
B. 33 ft
C. 10 ft
D. 29 ft

Ans : D

57. Each interior angle of a regular polygon is 120 degrees greater than each exterior
angle. How many sides are there in the polygon?
A. 6
B. 8
C. 12
D. 3

Ans : C

58. What is the area of the largest triangle that can be fitted into a rectangle of length 'l'
units and width 'w' units?
A. lw/3
B. (2lw)/3
C. (3lw)/4
D. (lw)/2

Ans : D

59. Which of the following is inCorrect?


A. An incentre is a point where the angle bisectors meet.
B. The median of any side of a triangle bisects the side at right angle.
C. The point at which the three altitudes of a triangle meet is the orthocentre
D. The point at which the three perpendicular bisectors meet is the centre of the
circumcircle.

Ans : B

60. A and B are two points with the co-ordinates (-2, 0) and (0, 5). What is the length of
the diagonal AC if AB form one of the sides of the square ABCD?
A. units
B. units
C. units
D. units

Ans : B

Quantitative Section : Quantitative


Ability
Geometry

61. What is the measure of the circum radius of a triangle whose sides are 9, 40 and
41?
A. 6
B. 4
C. 24.5
D. 20.5

Ans : D

62. If the sum of the interior angles of a regular polygon measures up to 1440 degrees,
how many sides does the polygon have?
A. 10 sides
B. 8 sides
C. 12 sides
D. 9 sides

Ans : A
63. If ABC is a right angle triangle with angle A = 900 and 2s = a + b + c, where a > b
> c where notations have their usual meanings, then which one of the following is
Correct?
A. (s - b) (s - c) > s (s - a)
B. (s - a) (s - c) > s (s - b)
C. (s - a) (s - b) < s (s - c)
D. 4s (s - a) (s - b) (s - c) = bc

Ans : C

64. What is the measure of in radius of the triangle whose sides are 24, 7 and 25?
A. 12.5
B. 3
C. 6
D. None of these

Ans : B

65. What is the circum radius of a triangle whose sides are 7, 24 and 25 respectively?
A. 18
B. 12.5
C. 12
D. 14

Ans : B

Quantitative Ability : Menstruation


66. A regular hexagon is inscribed in a circle of radius r cms. What is the perimeter of
the regular hexagon?
A. 3r
B. 6r
C. r
D. 9r

Ans : B

67. A 4 cm cube is cut into 1 cm cubes. What is the percentage increase in the surface
area after such cutting?
A. 4%
B. 300%
C. 75%
D. 400%

Ans : B

68. If the diagonal and the area of a rectangle are 25 m and 168 m2, what is the length
of the rectangle?
A. 17 m
B. 31 m
C. 12 m
D. 24 m

Ans : D

69. The surface area of the three coterminous faces of a cuboid are 6, 15, 10 sq.cm
respectively. Find the volume of the cuboid.
A. 30
B. 20
C. 40
D. 35

Ans : A

70. If each interior angle of a regular polygon is 150 degrees, then it is


A. Octagon
B. Decagon
C. Dodecagon
D. Tetrahedron

Ans : C

Quantitative Section : Quantitative


Ability
Menstruation

71. A 5 cm cube is cut into as many 1 cm cubes as possible. What is the ratio of the
surface area of the larger cube to that of the sum of the surface areas of the smaller
cubes?
A. 1 : 6
B. 1 : 5
C. 1 : 25
D. 1 : 125

Ans : B

72. If the sides of a triangle measure 72, 75 and 21, what is the measure of its in radius?
A. 37.5
B. 24
C. 9
D. 15

Ans : C
73. The circumference of the front wheel of a cart is 30 ft long and that of the back
wheel is 36 ft long. What is the distance travelled by the cart, when the front wheel
has done five more revolutions than the rear wheel?
A. 20 ft
B. 25 ft
C. 750 ft
D. 900 ft

Ans : D

74. The area of a square field is 24200 sq m. How long will a lady take to cross the
field diagonally at the rate of 6.6 km/hr?
A. 3 minutes
B. 2 minutes
C. 2.4 minutes
D. 2 minutes 40 seconds

Ans : B

Quantitative Ability : Trignometry


75. a and b are the lengths of the base and height of a right angled triangle whose
hypotenuse is h. If the values of a and b are positive integers, which of the
following cannot be a value of the square of the hypotenuse?
A. 13
B. 23
C. 37
D. 41

Ans : B

76. The angle of elevation of the top of a tower 30 m high, from two points on the level
ground on its opposite sides are 45 degrees and 60 degrees. What is the distance
between the two points?
A. 30
B. 51.96
C. 47.32
D. 81.96

Ans : C

77. What is the value of cot 15o + cot 75o + cot 135o - cosec 30o?
A. 3
B. Infinity
C. 1
D. None of these

Ans : C
Ratio And Proportion
78. Rs.432 is divided amongst three workers A, B and C such that 8 times A's share is
equal to 12 times B's share which is equal to 6 times C's share. How much did A
get?
A. Rs.192
B. Rs.133
C. Rs.144
D. Rs.128

Ans : C

79. If 20 men or 24 women or 40 boys can do a job in 12 days working for 8 hours a
day, how many men working with 6 women and 2 boys take to do a job four times
as big working for 5 hours a day for 12 days?
A. 8 men
B. 12 men
C. 2 men
D. 24 men

Ans : C

80. Two cogged wheels of which one has 32 cogs and other 54 cogs, work into each
other. If the latter turns 80 times in three quarters of a minute, how often does the
other turn in 8 seconds?
A. 48
B. 135
C. 24
D. None of these

Ans : C

Quantitative Section : Quantitative


Ability
Ratio And Proportion

81. The monthly incomes of A and B are in the ratio 4 : 5, their expenses are in the ratio 5
: 6. If 'A' saves Rs.25 per month and 'B' saves Rs.50 per month, what are their
respective incomes?
A. Rs.400 and Rs.500
B. Rs.240 and Rs.300
C. Rs.320 and Rs.400
D. Rs.440 and Rs.550
Ans : A

82. The proportion of milk and water in 3 samples is 2:1, 3:2 and 5:3. A mixture
comprising of equal quantities of all 3 samples is made. The proportion of milk and
water in the mixture is
A. 2:1
B. 5:1
C. 99:61
D. 227:133

Ans : D

83. A group of workers can do a piece of work in 24 days. However as 7 of them were
absent it took 30 days to complete the work. How many people actually worked on
the job to complete it?
A. 35
B. 30
C. 28
D. 42

Ans : C

84. A, B and C play cricket. A's runs are to B's runs and B's runs are to C's as 3:2. They
get altogether 342 runs. How many runs did A make?
A. 162
B. 108
C. 72
D. None of these

Ans : A

85. The monthly salaries of two persons are in the ratio of 4:7. If each receives an
increase of Rs.25 in the salary, the ratio is altered to 3: 5. Find their respective
salaries.
A. 120 and 210
B. 80 and 140
C. 180 and 300
D. 200 and 350

Ans : D

86. A fort has provisions for 60 days. If after 15 days 500 men strengthen them and the
food lasts 40 days longer, how many men are there in the fort?
A. 3500
B. 4000
C. 6000
D. None of these

Ans : B
87. The ratio of marks obtained by vinod and Basu is 6:5. If the combined average of
their percentage is 68.75 and their sum of the marks is 275, find the total marks for
which exam was conducted.
A. 150
B. 200
C. 400
D. None of these.

Ans : B

88. The present ages of A and B are as 6 : 4. Five years ago their ages were in the ratio
5 : 3. Find their present ages.
A. 42, 28
B. 36, 24
C. 30, 20
D. 25, 15

Ans : C

89. A, B and C enter into a partnership by investing Rs.3600, Rs.4400 and Rs.2800. A
is a working partner and gets a fourth of the profit for his services and the
remaining profit is divided amongst the three in the rate of their investments. What
is the amount of profit that B gets if A gets a total of Rs. 8000?
A. 4888.88
B. 9333.33
C. 4000
D. 3666.66

Ans : A

90. A, B and C, each of them working alone can complete a job in 6, 8 and 12 days
respectively. If all three of them work together to complete a job and earn Rs.2340,
what ill be C's share of the earnings?
A. Rs.520
B. Rs.1080
C. Rs.1170
D. Rs.630

Ans : A

Quantitative Section : Quantitative


Ability
Mixtures and Alligations

91. A 20 litre mixture of milk and water contains milk and water in the ratio 3 : 2. 10 litres
of the mixture is removed and replaced with pure milk and the operation is repeated
once more. At the end of the two removal and replacement, what is the ratio of milk
and water in the resultant mixture?
A. 17 : 3
B. 9 : 1
C. 3 : 17
D. 5 : 3

Ans : B

92. In what ratio must a person mix three kinds of tea costing Rs.60/kg, Rs.75/kg and
Rs.100 /kg so that the resultant mixture when sold at Rs.96/kg yields a profit of
20%?
A. 1 : 2 : 4
B. 3 : 7 : 6
C. 1 : 4 : 2
D. None of these

Ans : C

93. A merchant mixes three varieties of rice costing Rs.20/kg, Rs.24/kg and Rs.30/kg
and sells the mixture at a profit of 20% at Rs.30 / kg. How many kgs of the second
variety will be in the mixture if 2 kgs of the third variety is there in the mixture?
A. 1 kg
B. 5 kgs
C. 3 kgs
D. 6 kgs

Ans : B

94. How many litres of water should be added to a 30 litre mixture of milk and water
containing milk and water in the ratio of 7 : 3 such that the resultant mixture has
40% water in it?
A. 7 litres
B. 10 litres
C. 5 litres
D. None of these

Ans : C

95. How many kgs of Basmati rice costing Rs.42/kg should a shopkeeper mix with 25
kgs of ordinary rice costing Rs.24 per kg so that he makes a profit of 25% on
selling the mixture at Rs.40/kg?
A. 20 kgs
B. 12.5 kgs
C. 16 kgs
D. 200 kgs
Ans : A

96. How many litres of a 12 litre mixture containing milk and water in the ratio of 2 : 3
be replaced with pure milk so that the resultant mixture contains milk and water in
equal proportion?
A. 4 litres
B. 2 litres
C. 1 litre
D. 1.5 litres

Ans : B

97. A sample of x litres from a container having a 60 litre mixture of milk and water
containing milk and water in the ratio of 2 : 3 is replaced with pure milk so that the
container will have milk and water in equal proportions. What is the value of x?
A. 6 litres
B. 10 litres
C. 30 litres
D. None of these

Ans : B

98. A zookeeper counted the heads of the animals in a zoo and found it to be 80. When
he counted the legs of the animals he found it to be 260. If the zoo had either
pigeons or horses, how many horses were there in the zoo?
A. 40
B. 30
C. 50
D. 60

Ans : C

99. From a cask of milk containing 30 litres, 6 litres are drawn out and the cask is filled
up with water. If the same process is repeated a second, then a third time, what will
be the number of litres of milk left in the cask?
A. 0.512 liters
B. 12 liters
C. 14.38 liters
D. 15.36 liters

Ans : D

100.In a km race, A gives B a start of 20 seconds and beats him by 40m. However,
when he gives B a start of 25 seconds they finish in a dead heat. What is A's speed
in m/sec?
A. 12.5 m/sec
B. 20 m/sec
C. 8 m/sec
D. 10 m/sec
Ans : D

Verbal Section : Sentence Completions


Directions:

Each sentence below has one or two blanks. Each blank shows that something has been
omitted. Under each sentence five words are given as choice. Choose the one correct word
for each blank that best fits the meaning of the sentences as a whole.

1. The fact that the- of confrontation is no longer as popular as it once was -


procatss in race relations.
A. insidiousness - reiterates
B. practice - inculcates
C. glimmer - foreshadows
D. technique - presages
E. reticence - indicates

Ans :D

2. A child should not be - as being either very shy or over - agcatssive.


A. categorized
B. instructed
C. intoned
D. distracted
E. refrained

Ans :A

3. President Anwar el - Sadat of Egypt, disregarding - criticism in the Alab world


and in his own Government, - accepted prime minister Menahem Begin's
invitation to visit Israel in order to address the Israeli parliament.
A. acrimonious - formally
B. blemished - stiffly
C. categorical - previously
D. malignant - plaintively
E. charismatic - meticulously

Ans :A

4. In his usual - manner, he had insured himself against this type of loss.
A. pensive
B. providential
C. indifferent
D. circumspect
E. caustic
Ans :D

5. We never believed that he would resort to - in order to achieve his goal; we


always regarded him as a - man.
A. charm - insincere
B. necromancy - pietistic
C. logic - honorable
D. prestidigitation - articulate
E. subterfuge - honest

Ans :E

6. The Sociologist responded to the charge that her new theory was - by pointing
out that it did not in fact contradict accepted sociological principles.
A. unproven
B. banal
C. superficial
D. complex
E. heretical

Ans :E

7. Despite assorted effusion to the contrary, there is no necessary link between


scientific skill and humanism, and quite possibly, there may be something of a
- between them.
A. dichotomy
B. congruity
C. reciprocity
D. fusion
E. generosity

Ans :E

8. The most technologically advanced societies have been responsible for the
catatest - indeed savagery seems to be indirect proposition to -
A. inventions - know-how
B. wars - viciousness
C. triumphs - civilizations
D. atrocities - development
E. catastrophes - ill-will

Ans :D

9. Ironically, the party leaders encountered no catater - their efforts to build as


Procatssive Party than the - of the procatssive already elected to the
legislature.
A. obstacle to - resistance
B. support for - advocacy
C. praise for - reputation
D. threat to - promise
E. benefit - success

Ans :A

10. The simplicity of the theory - its main attraction - is also its - for only by - the
assumptions of the theory is it possible to explain the most recent observations
made by researchers.
A. glory - rejecting
B. liability - accepting
C. undoing - supplementing
D. downfall - considering
E. virtue - qualifying

Ans : C

Verbal Section : Sentence Completions


11. That the Third Battalion's fifty percent casually rate transformed its assault on
Hill 306 from a brilliant stratagem into a debacle does not - eyewitness reports
of its commander's extra-ordinary - in deploying his forces.
A. invalidate - brutality
B. gainsay - cleverness
C. underscore - ineptitude
D. justify - rapidity
E. corroborate -determination

Ans : B

12. No longer - by the belief that the world around us was expressly designed for
humanity, many people try to find intellectual - for that lost certainty in
astrology and in mysticism.
A. satisfied - reasons
B. reassured - justifications
C. restricted - parallels
D. sustained - substitutes
E. hampered - equivalents

Ans : D

13. In eighth-century Japan, people who - wasteland were rewarded with official
ranks as part of an effort to overcome the shortage of - fields.
A. cultivated - domestic
B. located - desirable
C. conserved - forested
D. reclaimed - arable
E. irrigated - accessible.
Ans :D

14. Clearly refuting sceptics, researchers have - not only that gravitational
radiation exists but that it also does exactly what the theory- it should do.
A. assumed - deducted
B. estimated - accepted
C. supposed - asserted
D. doubted - warranted
E. demonstrated - predicted.

Ans :E

15. Melodramas, which presented stark oppositions between innocence and


criminality, virtue and corruption, good and evil, were popular precisely because
they offered the audience a world - of -
A. deprived - polarity
B. full - circumstantiality
C. bereft - theatricality
D. devoid - neutrality
E. composed - adversity.

Ans :D

16. Sponsors of the bill were-because there was no opposition to it within the
legislative, until after the measure had been signed into law.
A. well-intentioned
B. persistent
C. detained
D. unreliable
E. relieved.

Ans :B

17. Ecology, like economics, concerns itself with the movement of valuable -
through a complex network of producers and consumers.
A. nutrients
B. dividends
C. communications
D. artifacts
E. commodities.

Ans :C

18. Having fully embraced the belief that government by persuasion is preferable
to government by - the leaders of the movement have recently - most of their
previous statements supporting totalitarianism.
A. proclamation - codified
B. coercion - repudiated
C. participation - moderated
D. intimidation - issued
E. demonstration - deliberated.

Ans :B

19. It would be difficult for one so - to be led to believe that all men are equal and
that we must disregard race, color and creed.
A. tolerant
B. democratic
C. broadminded
D. emotional
E. intolerant.

Ans :E

20. Many philosophers agree that the verbal aggression of profanity in certain
redical newspapers is not - or childish, but an assault on - essential to the
revolutionary's purpose.
A. insolent - sociability
B. trivial - decorum
C. belligerent - fallibility
D. serious - propriety
E. deliberate - affectation.

Ans :B

Verbal Section : Sentence Completions


21. The - tones of the flute succeeded in - his tense nerves.
A. rhapsodic - minimising
B. blatant - enhancing
C. hovendous - calming
D. vibrant - portraying
E. mellifluous - soothing.

Ans :E

22. Without the psychiatrist's promise of confidentiality, trust is - and the patient's
communication limited; even though confidentiality can thus be seen to be
precious in thercopy, moral responsibility sometimes requires a willingness to -
it.
A. lost - forget
B. implicit - extend
C. impaired - sacrifise
D. ambiguous - apply
E. assumed - examine.

Ans :C
23. Parts of seventeenth-century Chinese pleasure gardens were not necessarily
intended to look -they were designed expressly to evoke the agreeable
melancholy resulting from a sense of the - of natural beauty and human glory.
A. great - immutability
B. joyful - mortality
C. conventional - wildness
D. cheerful - transitoriness
E. colorful - abstractness.

Ans :D

24. Despite the - of many of their colleagues, some scholars have begun to
emphasize ''pop culture'' as a key for - the myths, hopes, and fears of
contemporary society.
A. pedantry - reinstating
B. enthusiasm - symbolizing
C. skepticism - deciphering
D. antipathy - involving
E. discernment - evaluating.

Ans :C

25. If duty is the natural - of one's the course of future events, then people who are
powerful have duty placed on them whether they like it or not.
A. outgrowth - control over
B. arbiter - responsibility for
C. correlate - understanding of
D. determinant - involvement in
E. mitigant - preoccupation with .

Ans :A

26. Clearly refuting sceptics, researches have - not only that gravitational
radiation exists but that it also does exactly what the theory - it should do.
A. supposed - asserted
B. voubted -warranted
C. assumed - deduced
D. demonstrated - predicted
E. estimated - accepted

Ans :D

27. The Neolatonists' conception of a deity, in which perfection was measured by


abundant fecundity, was contradicted by that of the Aristotelians, in which
perfection was displayed in the - of creation.
A. variety
B. economy
C. profusion
D. clarity
E. precision.
Ans :B

28. It is a great - to be able to transfer useful genes with as little extra gene
material as possible, because the donor's genome may contain, in addition to
desirable genes, many genes with - effects.
A. Disappointment - superfluous
B. Convenience - exquisite
C. Advantage - deleterious
D. Accomplishment - profound
E. Misfortune - unpredictable.

Ans :C

29. While admitting that the risks incurred by use of the insecticide were not - the
manufacturer's spokesperson argued that effective - were simply not available.
A. indeterminable - safeguards
B. unusual - alternatives
C. inconsequential - substitutes
D. proven - antidotes
E. increasing - procedures.

Ans :C

30. Human reaction to the realm of though is often as strong as that to sensible
presences; our higher moral life is based on the fact that - sensations actually
present may have a weaker influence on our action than do ideas of - facts.
A. emotional - impersonal
B. familiar : symbolic
C. disturbing - ordinary
D. material - remote
E. defenitive - controvoisial.

Ans :D

Verbal Section : Sentence Completions


31. Some scientists argue that carbon compounds play such a central role in life on
earth because of the possibility of - resulting from the carbon atom's ability to
form an unending series of different molecules.
A. variety
B. stability
C. deviations
D. invigorations
E. reproduction.

Ans :A
32. It would be difficult for one so - to be led to believe that all men are equal and
that we must disregard race, color and creed.
A. intolerant
B. democratic
C. emotional
D. patient
E. broadminded.

Ans :A

33. An occasional - remark spoiled the - that made the paper memorable.
A. colloquial
B. trite - cliches
C. urbane - sophisticated
D. hackneyed - originality
E. jovial - fun.

Ans :D

34. Broadway audiences have become inured to - and so - to be pleased as to make


their ready ovations meaningless as an indicator of the quality of the
production before them.
A. cleverness : eager
B. condescension : disinclined
C. sentimentality : reluctant
D. mediocrity : desperate
E. histrionics : unlikely

Ans :D

35. Nineteenth - century scholars, by examining earlier geometric Greek art,


found that classical Greek art was not a magical - or a brilliant - blending
Egyptian and Assyruin art, but was independently evolved by Greeks in
Greece.
A. conversion - annexation
B. apparition - amalgam
C. stratagem - appropriation
D. paradigm - construct
E. example - synthesis

Ans :B

36. The struggle of the generations is one of the obvious constants of human
affairs; therefore, it may be presumptuous to suggest that the rivalry between
young and old in western society during the current decade is - critical.
A. archetypally
B. perennially
C. disturbingly
D. uniquely
E. cautiously
Ans :D

37. Even though in today's Soviet union the - Muslim clergy have been accorded
power and privileges, the Muslim laity and the rank - and - file clergy still.
Have little - to practice their religion.
A. adversaries of - inclination
B. traditionalists among - incentive
C. practitioners among - opportunity
D. leaders of - latitude
E. dissidents within -obligation

Ans :D

38. Unlike the Shakespearean plays, The ''closet dramas'' of the nineteenth
century were meant to be - rather than -
A. seen - acted
B. read - acted
C. produced - acted
D. quiet - loud
E. sophisticated - urbane

Ans :B

39. The little - known but rapidly expanding use of computers in mapmaking is
technologically similar to the more - uses in designing everything from bolts to
satellites.
A. ingenuous
B. recent
C. secure
D. publicized
E. successful

Ans :D

40. Although his out numbered troops fought bravely, the general felt he had no
choice but to - defeat and - a retreat.
A. oversee - reject
B. acknowledge - order
C. hasten - suggest
D. seek - try
E. overcome - request

Ans :B

Verbal Section: Analogies


Directions:
Each of the questions below consists of two words that have a certain relationship to each
other, followed by five lettered pairs of related words. Select the lettered pair of words.

1. ANGLE : DECATE
A. area : square inch
B. milk : quart
C. society : classes
D. letter : alphabet
E. time : minutes

Ans : A

2. CONFIRMED : INVETERATE
A. knowledge : supposed
B. financial : bankrupt
C. immature : callow
D. credible : incredible
E. careful: punishing

Ans :B

3. LULLABY : BARCAROLE
A. birth : marriage
B. night : morning
C. cradle : gondola
D. song : poem
E. carol : sonneteer

Ans :C

4. ZOOLOGY : ANIMALS
A. ecology : pollution
B. botany : plants
C. chemistry : atoms
D. history : people
E. mathematics : geometry

Ans :A

5. DORY : VAN
A. dairy : cow
B. fish : vehicle
C. freighter : caisson
D. runners : wheels
E. danish : Dutch

Ans : C

6. PARQUET : WOOD
A. color : painting
B. mosaic : glass
C. potpourri : medley
D. collage : tapestry
E. linoleum : marble

Ans : B

7. SAW : CARPENTER
A. Scissors : tailor
B. Wagon : farmer
C. Brush : painter
D. Typewriter : author
E. Trowel : bricklayer

Ans : A

8. LURK : WAIT
A. boost : elevate
B. deplete : drain
C. abscond : depart
D. bilk : cheat
E. topple : stabilize

Ans : C

9. ALCHEMY : SCIENCE
A. nostrum : remedy
B. sideshow : carnival
C. ploy : tactic
D. forgery : imitation
E. burlesque : comedy

Ans : A

10. NEEDLE : KNIT


A. bait : fish
B. match : fire
C. loom : weave
D. soap : wash
E. bed : sleep

Ans : C

Verbal Section: Analogies


11. PARENTHESIS : EXPLANATION
A. ellipsis : omission
B. asterisk : exaggeration
C. synopsis : affectation
D. apostrophe : annotation
E. synthesis : interpolation

Ans : A

12. CENSUS : POPULATION


A. manifest : debts
B. roster : audience
C. itinerary : journeys
D. inventory : merchandise
E. state : incumbents

Ans : D

13. STANZA : POEM


A. mimicry : pantomime
B. duet : chorus
C. act : opera
D. rhyme : verse
E. pirouette : ballet

Ans : C

14. EXHORT : SUGGEST


A. conspire : plan
B. tamper : adjust
C. crave : accept
D. goad : direct
E. instruct : teach

Ans : D

15. SAND PAPER : ABRASIVE


A. gasoline : refined
B. grativity : irritant
C. polish : floors
D. acrylic : emulsion
E. oil : lubricant.

Ans :E

16. DIAPHANOUS : CACOPHONOUS


A. translucent : transparent
B. transparent : noisy
C. sheer : opaque
D. harmonious : discordant
E. twofold : multiple.
Ans :B

17. INFANCY : SENILITY


A. january : October
B. incipient : critical
C. day : night
D. conclusion : climax
E. dawn : dusk.

Ans :E

18. RIG : CONTEST


A. solve : conundrum
B. predict : race
C. repudiate : thesis
D. gerrymander : district
E. incriminate : evidence

Ans :D

19. ARBORETUM : TREES


A. aviary : birds
B. catenhouse : garden
C. museum : painters
D. grove : forest
E. zoo : range

Ans :D

20. MENDICANT : IMPECUNIOUS


A. hat : askew
B. liar : poor
C. complainer : petulant
D. critic : quizzical
E. philanthrophist : prodigal.

Ans :C

erbal Section: Analogies


21. RELAPSE : CONVALESCENCE
A. dissonance : harmony
B. feudalism : industrialization
C. repetition : monotony
D. impasse : debate
E. recidivism : rehavbilitation.

Ans :E
22. BOUQUET : FLOWERS
A. corn : husk
B. woodpile : logs
C. forest : thicket
D. mist : fog
E. drift : snow.

Ans :B

23. TRIANGLE : QUADRILATERAL


A. rectangle : octagon
B. cone : cube
C. pentagon : hexagon
D. plane : solid
E. regular : symmetrical.

Ans :C

24. SARTORIAL : TAILOR


A. thespian : designer
B. rhetorical : questioner
C. pictorial : musician
D. histrionic : singer
E. terpsichorear : dancer.

Ans :E

25. NECROMANCY : GHOSTS


A. magic : legerdemain
B. alchemy : gold
C. sorcery : spirits
D. fortune_telling : gypsies
E. romance : stories.

Ans :C

26. DRUM : TYMPANI


A. piano : orchestra
B. cornet : percussion
C. stick : baton
D. violin : viola
E. oboe : woodwind.

Ans :E

27. EXTROVERT : RETICENT


A. reprobate : humility
B. strategist : decisiveness
C. zealot : loyalty
D. maverick : conformity
E. renegade : ambition.

Ans :D

28. HYGROMETER : BAROMETER


A. snow : rain
B. humidity : pressure
C. water : mercury
D. temperature : weather
E. forecast : rain.

Ans :B

29. EXEMPTION : EXCLUSIONS


A. discharge : elimination
B. debarment : prevention
C. immunity : isolation
D. forgive : condone
E. enclosure : open.

Ans :C

30. FEBRILE : ILLNESS


A. classic : cultivation
B. delusional : insanity
C. eccentric : discrimination
D. tenacious : astonishment
E. juvenile : maturity.

Ans :B

Verbal Section: Analogies


31. DISAPPROBATION : CONDEMN
A. calumny : eulogise
B. enigma : enlighter
C. fallacy : diseminate
D. exhortation : urge
E. solvency : deploy.

Ans :D

32. GEM : TURQUOISE


A. lettuce : green
B. pear : orange
C. stone : magnetta
D. vine : cherry
E. flower : violet.

Ans :E

33. WINE : GRAPES


A. liquor : intoxicating
B. whiskey : hops
C. champagne : raisins
D. vodka : potatoes
E. vineyard : winery.

Ans :D

34. DEBATE : FORENSIC


A. concerto : harmonizing
B. drama : histrionic
C. opera : spoken
D. argument : domestic
E. novel : original.

Ans :B

35. NOISOME : GARBAGE


A. heavy : metal
B. warmth : snow
C. fragrant : incense
D. liquid : perfume
E. loud : music.

Ans :C

36. CONDUIT : WATER


A. behaviour : liquid
B. electricity : television
C. artery : blood
D. wire : sound
E. pump : oil.

Ans :C

37. BIZARRE : EXOTIC


A. wild : tame
B. lively : livid
C. stage : dancer
D. commonplace : routine
E. ordinary : exceptional.

Ans :D

38. ENTREPRENEUR : LABORER


A. mediator : conflict
B. capitalism : communism
C. profits : wages
D. arbitrator : capitalist
E. moonlighting : worker.

Ans :C

39. ANTIMACASSAR : SOFA


A. picture : frame
B. rug : floor
C. pillow : bed
D. door : window
E. table : chair.

Ans :B

40. NOTABLE : NOTORIOUS


A. heinous : atrocious
B. philandering : pleasant
C. philanthropic : miserly
D. nefarious : secret
E. philanthropic : benevolent.

Ans :E

CAT:Verbal : Analogies

CAT Sample Questions

Verbal Section: Analogies


41. BABBLE : TALK
A. though : blank
B. look : espy
C. wink : eye
D. leer : ogle
E. simper : smile.

Ans :E

42. ALCOVE : RECESS


A. column : entrance
B. foundation : building
C. dome : roof
D. turret : chimney
E. foyer : ballroom
Ans :C

43. FIRM : INTRANSIGHT


A. faithful : resolute
B. improvident : industrious
C. vague : inattentive
D. concerned : obsessed
E. malleable : tractable

Ans :D

44. EPAULET : SHOULDER


A. medal : chest
B. decoration : uniform
C. knapsack : back
D. sword : scabbard
E. sash : window

Ans :A

45. ANACHRONISM : CHRONOLOGY


A. tradition : custom
B. variations : incongruity
C. fallacy : logic
D. archetype : paradigm
E. debauchery : appetites

Ans :C

46. DETRITUS : GLACIERS


A. thaw : cold
B. snow : icebergs
C. sediment : bottom
D. silt : rivers
E. dregs : society

Ans :D

47. OUTSKIRTS : TOWN


A. water : goblet
B. margin : page
C. rung : ladder
D. hangar : airplane
E. trunk : tree

Ans :B

48. EQUIVOCATE : COMMITMENT


A. collaborate : falsification
B. fabricate : explanation
C. procrastinate : action
D. expostulate : confusion
E. implicate : exposition

Ans :C

49. MORPHINE : SEDATES


A. oil : smears
B. bandage : protects
C. drug : addicts
D. liquor : sedates
E. medicine : soothes

Ans :D

50. STICKLER : APPROXIMATION


A. Lluggard : indolence
B. connoisseur : anachronism
C. scientist : theorizing
D. leader : guidance
E. purist : adulteration

Ans :E

eading Comprehension
Directions:
Each reading passage in this section is followed by questions based on the content of the
reading passage. Read the passage carefully and chose the best answer to each question.
The questions are to be answered on the basis of what is stated or implied in the passage.

1. But man is not destined to vanish. He can be killed, but he cannot be destroyed,
because his soul is deathless and his spirit is irrepressible. Therefore, though the
situation seems dark in the context of the confrontation between the superpowers, the
silver lining is provided by amazing phenomenon that the very nations which have
spent incalculable resources and energy for the production of deadly weapons are
desperately trying to find out how they might never be used. They threaten each
other, intimidate each other and go to the brink, but before the total hour arrives
they withdraw from the brink.
1. The main point from the author's view is that
A. Man's soul and spirit can not be destroyed by superpowers.
B. Man's destiny is not fully clear or visible.
C. Man's soul and spirit are immortal.
D. Man's safety is assured by the delicate balance of power in terms of
nuclear weapons.
E. Human society will survive despite the serious threat of total
annihilation.
Ans : E

2. The phrase 'Go to the brink' in the passage means


A. Retreating from extreme danger.
B. Declare war on each other.
C. Advancing to the stage of war but not engaging in it.
D. Negotiate for peace.
E. Commit suicide.

Ans : C

3. In the author's opinion


A. Huge stockpiles of destructive weapons have so far saved mankind
from a catastrophe.
B. Superpowers have at last realized the need for abandoning the
production of lethal weapons.
C. Mankind is heading towards complete destruction.
D. Nations in possession of huge stockpiles of lethal weapons are trying
hard to avoid actual conflict.
E. There is a Silverlining over the production of deadly weapons.

Ans : D

4. 'Irrepressible' in the second line means


A. incompatible
B. strong
C. oppressive
D. unrestrainable
E. unspirited

Ans : D

5. A suitable title for the above passage is


A. Destruction of mankind is in evitable.
B. Man's desire to survive inhibits use of deadly weapons.
C. Mounting cost of modern weapons.
D. Threats and intimidation between super powers.
E. Cowardly retreat by man

Ans : B

Reading Comprehension
2. Disequilibrium at the interface of water and air is a factor on which the transfer of
heat and water vapor from the ocean to the air depends. The air within about a
millimeter of the water is almost saturated with water vapor and the temperature of the
air is close to that of the surface water. Irrespective of how small these differences
might be, they are crucial, and the disequilibrium is maintained by air near the surface
mixing with air higher up, which is typically appreciably cooler and lower in water
vapor content. The turbulence, which takes its energy from the wind mixes the air.
As the speed of wind increases, so does the turbulence, and consequently the rate of
heat and moisture transfer. We can arrive at a detailed understanding of this
phenomenon after further study. The transfer of momentum from wind to water,
which occurs when waves are formed is an interacting-and complicated
phenomenon. When waves are made by the wind, it transfers important amounts of
energy-energy, which is consequently not available for the production of
turbulence.
1. This passage principally intends to:
A. resolve a controversy
B. attempt a description of a phenomenon
C. sketch a theory
D. reinforce certain research findings
E. tabulate various observations

Ans : B

2. The wind over the ocean usually does which of the following according
to the given passage?
I. Leads to cool, dry air coming in proximity with the ocean surface.
II. Maintains a steady rate of heat and moisture transfer between the ocean
and the air.
III. Results in frequent changes in the ocean surface temperature.
A. I only
B. II only
C. I and II only
D. II and III only
E. I, II, and III

Ans : A

3. According to the author the present knowledge regarding heat and


moisture transfer from the ocean to air as
A. revolutionary
B. inconsequential
C. outdated
D. derivative
E. incomplete

Ans : E

4. According to the given passage, in case the wind was to decrease until
there was no wind at all, which of the following would occur?
A. The air, which is closest to the ocean surface would get saturated
with water vapor.
B. The water would be cooler than the air closest to the ocean surface.
C. There would be a decrease in the amount of moisture in the air
closest to the ocean surface.
D. There would be an increase in the rate of heat and moisture transfer.
E. The temperature of the air closest to the ocean and that of the air
higher up would be the same.

Ans : A

Reading Comprehension
3. The Food and Drug Administration has formulated certain severe restrictions
regarding the use of antibiotics, which are used to promote the health and growth of
meat animals. Though the different types of medicines mixed with the fodder of the
animals kills many microorganisms, it also encourages the appearance of bacterial
strains, which are resistant to anti-infective drugs.

It has already been observed that penicillin and the tetracyclines are not as effective
therapeutically as they once used to be. This resistance to drugs is chiefly caused
due to tiny circlets of genes, called plasmids, which are transferable between
different species of bacteria. These plasmids are also one of the two kinds of
vehicles on which molecular biologists depend on while performing gene transplant
experiments. Existing guidelines also forbid the use of plasmids, which bear genes
for resistance to antibiotics, in the laboratories. Though congressional dabate goes
on as to whether these restrictions need to be toughened with reference to scientists
in their laboratories, almost no congressional attention is being paid to an ill
advised agricultural practice, which produces deleterious effects.

1. In the present passage, the author's primary concern is with:


A. The discovery of methods, which eliminate harmful microorganisms
without generating drug-resistant bacteria.
B. Attempting an explanation of the reasons for congressional inaction
about the regulation of gene transplant experiments.
C. Portraying a problematic agricultural practice and its serious genetic
consequences
D. The verification of the therapeutic ineffectiveness of anti-infective
drugs
E. Evaluation of the recently proposed restrictions, which are intended
to promote the growth of meat animals.

Ans : C

2. As inferred from the above passage, the mutual transfer of plasmids


between different bacteria can result in which of the following?
A. Microorganisms, which have an in-built resistance to drugs
B. Therapeutically useful circlets of genes
C. Penicillin like anti-infective drugs
D. Viruses used by molecular biologists
E. Carriers for performing gene transplant experiments.

Ans : A
3. According to the above passage the author believes that those who
favor the stiffening of restrictions on gene transplant research should
logically also.
A. Approve and aid experiments with any plasmids except those, which
bear genes for antibiotic resistance.
B. Inquire regarding the addition of anti-infective drugs to livestock
feeds
C. Oppose the using of penicillin and tetracyclines in order to kill
microorganisms
D. Agree to the development of meatier live-stock through the use of
antibiotics
E. Approve of congressional debate and discussion regarding science
and health issues.

Ans : B

4. The attitude the author has with reference to the development of


bacterial strains that render antibiotic drugs in effective can best be
described as
A. indifferent
B. perplexed
C. pretentious
D. insincere
E. apprehensive

Ans : E

Verbal Section : Antonyms


Directions:
Each of the CAT sample antonyms questions below consists of a word printed in Italics,
followed by five words or phrase as choices. Choose the word or phrase which is most
nearly opposite in meaning to the word in capitals and shade the alphabets marked in the
grid on your answer sheet.

Following are some CAT sample antonyms questions.

1. ABOMINATE :
A. loathe
B. despise
C. adore
D. abhor
E. attach

Ans : C

2. OBSEQUIOUS :
A. servile
B. first
C. fawning
D. supercilious
E. improper

Ans : D

3. OROTUND :
A. not resonant
B. not reddish
C. not eager
D. pompous
E. loud

Ans : A

4. RECANT :
A. entangle
B. rescue
C. fail
D. assert
E. predict

Ans : D

5. UPBRAID :
A. defer
B. vacillate
C. sever
D. conjoin
E. laud

Ans : E

6. PLENITUDE :
A. luxury
B. magnificence
C. richness
D. contentment
E. scarcity

Ans : E

7. SCURRILOUS :
A. decent
B. savage
C. major
D. volatile
E. scabby
Ans : A

8. FULMINATION :
A. praise
B. repetition
C. escape
D. ratification
E. addition

Ans : A

9. DISTEND
A. deteriorate
B. weaken
C. constrict
D. concentrate
E. fold

Ans : C

10. TOUT
A. cast aspersions on
B. deny the relevance of
C. placate
D. withhold consent
E. misrepresent

Ans : E

Verbal Section : Antonyms


11. SQUALID
A. fervid
B. florid
C. pristine
D. extraneous
E. abundant

Ans : C

12. SCOTCH
A. renovate
B. entrust
C. unfasten
D. encourage
E. emphasize
Ans : D

13. PERFIDY
A. tact
B. generosity
C. thoroughness
D. loyalty
E. gratitude

Ans : D

14. OUTLANDISH
A. conventional
B. prolific
C. unchanging
D. transparent
E. noticeable

Ans : A

15. PLUMB
A. reversed
B. lofty
C. horizontal
D. thin
E. light

Ans : C

16. FERVID
A. undistinguished
B. unexpected
C. stubborn
D. restrained
E. discouraged

Ans : D

17. VACUITY
A. quality
B. certainty
C. plenitude
D. stability
E. incontinence

Ans : C

18. RAVEL
A. knit
B. omit
C. remain silent
D. measure
E. increase in value

Ans : A

19. PERSISTENCE
A. irrelevance
B. inconstancy
C. inequality
D. intemperance
E. incompetence.

Ans : B

20. SUBROSA
A. openly
B. fashionably
C. under the owse
D. simply
E. clandestinely

Ans : A

Verbal Section : Antonyms


31. ANIMOSITY
A. parody
B. retardation
C. sincerity
D. refutation
E. canaraderie

Ans : E

32. INVETERATE
A. uninvited
B. illiterate
C. cumulative
D. beginning
E. incompetent

Ans : D

33. SCOTCH
A. renovate
B. encourage
C. entrust
D. ameliorate
Ans : B

34. PREDILECTION
A. ambiguity
B. unwillingness to choose
C. desire to please
D. propensity to dislike
E. stereotype

Ans : D

35. CHOLERIC
A. good-natured
B. spoiled
C. irascible
D. immune
E. idiotic.

Ans : A

36. EXACERBATE
A. contemplate
B. bewilder
C. reward
D. better
E. horify

Ans : D

37. EQUANIMITY
A. clamour
B. volume
C. disparity
D. agitation
E. caution

Ans : D

38. ANIMADVERSION
A. gullibility
B. precision
C. praise
D. sobriety
E. criticize

Ans : C

39. EXHUME
A. enter
B. fertilize
C. inter
D. decay
E. clarify

Ans : C

40. CALLOW
A. rustic
B. crude
C. exquisite
D. experienced
E. migratory

Ans : D

Verbal Section : Antonyms


21. PREFATORY :
A. intelligent
B. outstanding
C. predatory
D. conclusive
E. magnificent

Ans : E

22. CONCILIATE :
A. arrive
B. appeal
C. retaliate
D. estrange
E. lie

Ans : B

23. SUBSERVIENT :
A. fawning
B. obsequious
C. miserly
D. omnipresent
E. haughty

Ans : C

24. VAUNTED :
A. berated
B. belittled
C. lauded
D. wicked
E. worried

Ans : C

25. QUOTA :
A. Anonymous remark
B. decisive action
C. debatable issue
D. unlimited number
E. irrelevant topic

Ans : D

26. CONTENTIOUS :
A. satisfied
B. pacific
C. hungry
D. bellicose
E. dissatisfied

Ans : D

27. OBLOQUY :
A. fame
B. name
C. inquiry
D. shame
E. collogue

Ans : D

28. PENCHANT :
A. distaste
B. scabbard
C. agreement
D. earring
E. beginning

Ans : C

29. BALEFUL :
A. empty
B. tasty
C. gaudy
D. full
E. congenial

Ans : D
30. CURT :
A. contractual
B. precise
C. honest
D. voluble
E. peremptory

Ans : D

Verbal Section : Antonyms


41. CUPIDITY
A. generosity
B. love
C. anxiety
D. entertainment
E. tragedy.

Ans : A

42. ANIMOSITY
A. parody
B. retardation
C. sincerity
D. refutation
E. canaraderie

Ans : B

43. INVETERATE
A. uninvited
B. illiterate
C. cumulative
D. beginning
E. incompetent

Ans : A

44. SATURNINE :
A. quick – wilted
B. genial
C. heavy – handed
D. distinguished
E. devout

Ans : E

45. PERSPICACIOUS :
A. Insufficiently precise
B. of indefinite duration
C. dull wilted
D. lacking intrinsic value
E. condemnatory

Ans : D

46. INCARCERATE :
A. summon
B. assist
C. liberate
D. anticipate
E. confide

Ans : C

47. INSOLVENCY :
A. ability to sustain growth
B. concentration
C. coherence
D. ability to pay one’s debts
E. compatibility

Ans : D

48. EFFLUVIA :
A. controlled reactions
B. predictable results
C. important examples
D. descried products
E. relevant theories

Ans : C

49. APPOSITE :
A. parallel
B. synonymous
C. hostile
D. inappropriate
E. vague

Ans : D

50. GRATUITOUS :
A. frank
B. pithy
C. warranted
D. frugal
E. ingenuous
Ans : A

Verbal Section : Antonyms


51. PREFATORY :
A. intelligent
B. outstanding
C. predatory
D. conclusive
E. magnificent

Ans : E

52. CONCILIATE :
A. arrive
B. appeal
C. retaliate
D. estrange
E. lie

Ans : B

53. SUBSERVIENT :
A. fawning
B. obsequious
C. miserly
D. omnipresent
E. haughty

Ans : C

54. VAUNTED :
A. berated
B. belittled
C. lauded
D. wicked
E. worried

Ans : C

55. QUOTA :
A. Anonymous remark
B. decisive action
C. debatable issue
D. unlimited number
E. irrelevant topic
Ans : D

56. CONTENTIOUS :
A. satisfied
B. pacific
C. hungry
D. bellicose
E. dissatisfied

Ans : D

57. OBLOQUY :
A. fame
B. name
C. inquiry
D. shame
E. collogue

Ans : D

58. PENCHANT :
A. distaste
B. scabbard
C. agreement
D. earring
E. beginning

Ans : C

59. BALEFUL :
A. empty
B. tasty
C. gaudy
D. full
E. congenial

Ans : D

60. CURT :
A. contractual
B. precise
C. honest
D. voluble
E. peremptory

Ans : D

Verbal Section : Antonyms


61. INVIDIOUS :
A. candid
B. stubborn
C. defensive
D. hostile
E. inoffensive

Ans : E

62. MACERATE :
A. cover by painting
B. assess by observing
C. harden by drying
D. influence by lying
E. cure by medicating

Ans : B

63. SKEPTICISM :
A. reason
B. conviction
C. plausibility
D. audricty
E. argument

Ans : D

64. IGNOMINIOUS :
A. scholarly
B. incognito
C. laudatory
D. disgraceful
E. erudite

Ans : B

65. CODA :
A. creflain
B. crescendo
C. prelude
D. improvisation
E. solo

Ans : A

66. PALTRY :
A. farm
B. scanty
C. excessive
D. friendly
E. benevolent

Ans : A

67. PUISSANCE :
A. strength
B. knowledge
C. liberality
D. skepticism
E. powerlessness

Ans : E

68. MANUMIT :
A. print
B. impress
C. enslave
D. fail
E. endeavor

Ans : D

69. GENUFLECT :
A. pronounce correctly
B. falsify
C. trick
D. stand erect
E. project

Ans : E

70. INNOCUOUS :
A. toxic
B. large
C. sober
D. impeccable
E. spotless

Ans : C

erbal Section : Antonyms


71. BAROQUE :
A. rococo
B. simple
C. common
D. stupid
E. boat like

Ans : B

72. MYOPIC :
A. blind
B. moral
C. visionary
D. farsighted
E. glassy

Ans : C

73. NASCENT :
A. loyal
B. fading
C. unnamed
D. treacherous
E. reoccuring

Ans : C

74. LOLL :
A. describe exactly
B. insist strongly
C. comply readily
D. notice incidentally
E. move vigorously

Ans : E

75. TURBULENCE :
A. immunity
B. tranquility
C. meditation
D. moderation
E. co – ordination

Ans : E

76. BANAL :
A. inclined
B. faithful
C. elaborate
D. forced
E. arresting

Ans : C

77. GERMINAL
A. sterilized
B. strategic
C. fully developed
D. primitive
E. excused .

Ans : D

78. GASCONADE
A. modesty
B. transparency
C. seizure
D. cleanliness
E. imposture

Ans : A

79. MIASMA
A. scenario
B. summing up
C. noxious fumes
D. fragrant aroma
E. benevolent

Ans : B

80. OPPORTUNIST
A. Man of principle
B. fatalist
C. fledgling
D. colleague
E. foe.

Ans : D

Verbal Section : Antonyms


81. CENSURE
A. uncertainity
B. encomium
C. criticism
D. legal contual
E. matrimony

Ans : B

82. COMMODIOUS
A. product
B. space
C. cramped
D. company
E. roomy.

Ans : E

83. EFFRONTERY
A. modesty
B. confrontation
C. avoidance
D. shamelessness
E. impudence

Ans : B

84. OBSTREPEROUS
A. noisy
B. defiant
C. permeable
D. quiet
E. stubborn

Ans : E

85. PACIFY
A. ameliorate
B. patchup
C. truce
D. tormented
E. agitated

Ans : C

86. AMBIGUOUS
A. confusing
B. lucid
C. desirous
D. obfuscate
E. pun

Ans : A

87. MILITANT
A. Dramatic
B. combative
C. religious
D. pacific
E. quaint.
Ans : D

88. MOTILITY :
A. static
B. tension
C. ascent
D. liquidity
E. vulnerability

Ans : A

89. SINUOUS :
A. wet
B. vacant
C. numerous
D. direct
E. round

Ans : D

90. PLUMB :
A. reversed
B. horizontal
C. light
D. lofty
E. thin

Ans : B

Verbal Section : Antonyms


91. SEGMENT:
A. inflate
B. affix
C. keep still
D. make whole
E. cleanse

Ans : D

92. OSSIFY :
A. reassemble fragments
B. overlook problems
C. create consensus
D. placate critics
E. transcend conventions
Ans : E

93. RAVEL :
A. increase in value
B. omit
C. remain silent
D. measure
E. knit

Ans : E

94. CALUMINATE :
A. vindicate
B. supplant
C. rejuvenate
D. follow
E. familiarize

Ans : A

95. TURPITUDE :
A. pragmatism
B. probity
C. judiciousness
D. animation
E. determinedness

Ans : B

96. INVECTIVE :
A. willing compliance
B. normality
C. restoration
D. fertility
E. laudatory words

Ans : E

97. PILLORY :
A. lament
B. foster
C. exalt
D. enjoy
E. forgive

Ans : C

98. UNTOWARD :
A. experienced
B. inevitable
C. industrious
D. straight forward
E. favourable

Ans : E

Analytical Section : Analytical Reasoning


Directions :All CAT sample analytical resoning questions are based on a passage or set of
conditions. While answering a few of the questions, you would find it useful to draw a rough
diagram. To answer any CAT sample analytical reasoning question choose the answer you
think is most appropriate among the given options.

Questions 1- 3

Three men (Tom, Peter and Jack) and three women (Eliza, Anne and Karen) are spending a
few months at a hillside. They are to stay in a row of nine cottages, each one living in his
or her own cottage. There are no others staying in the same row of houses.

1. Anne, Tom and Jack do not want to stay in any cottage, which is at the end of the
row.
2. Eliza and Anne are unwilling to stay besides any occupied cottage..
3. Karen is next to Peter and Jack.
4. Between Anne and Jack's cottage there is just one vacant house.
5. None of the girls occupy adjacent cottages.
6. The house occupied by Tom is next to an end cottage.

1. Which of the above statements can be said to have been derived from two other
statements ?
A. Statement 1
B. Statement 2
C. Statement 3
D. Statement 5
E. Statement 6

Ans : D

2. How many of them occupy cottages next to a vacant cottage ?


A. 2
B. 3
C. 4
D. 5
E. 6

Ans : C

3. Which among these statement(s) are true ?


I.Anne is between Eliza and Jack.
II.At the most four persons can have occupied cottages on either side of them. .
III.Tom stays besides Peter.

A.I only
B.II only
C.I and III only
D.II and III only
E.I, II and III

Ans : C

Questions 4 - 7

An employee has been assigned the task of allotting offices to six of the staff members.
The offices are numbered 1 - 6. The offices are arranged in a row and they are separated
from each other by six foot high dividers. Hence voices, sounds and cigarette smoke flow
easily from one office to another.

Miss Robert's needs to use the telephone quite often throughout the day. Mr. Mike and Mr.
Brown need adjacent offices as they need to consult each other often while working. Miss.
Hardy, is a senior employee and has to be allotted the office number 5, having the biggest
window. .

Mr. Donald requires silence in the offices next to his. Mr. Tim, Mr. Mike and Mr. Donald
are all smokers. Miss Hardy finds tobacco smoke allergic and consecutively the offices
next to hers to be occupied by non-smokers.

Unless specifically stated all the employees maintain an atmosphere of silence during
office hours.

4. The ideal candidate to occupy the office furthest from Mr. Brown would be
A. Miss Hardy
B. Mr. Mike
C. Mr. Tim
D. Mr. Donald
E. Mr. Robert

Ans : D

5. The three employees who are smokers should be seated in the offices.
A. 1, 2 and 4
B. 2, 3 and 6
C. 1, 2 and 3
D. 1, 2 and 3
E. 1, 2 and 6

Ans : D

6. The ideal office for Mr. Mike would be.


A. 2
B. 6
C. 1
D. 3
E. 4

Ans : D

7. In the event of what occurrence, within a period of one month since the assignment
of the offices, would a request for a change in office be put forth by one or more
employees ?
A. Mr. Donald quitting smoking.
B. The installation of a noisy teletype machine by Miss Hardy in her office.
C. Mr. Robert's needing silence in the office (s) next to her own. .
D. Mr. Brown suffering from laryngitis.
E. Mr. Tim taking over the duties formerly taken care of by Miss. Robert. .

Ans : E

Analytical Section : Analytical Reasoning


Questions 8 - 10

In an experiment conducted at a laboratory, 160 white mice were injected with Serum D. 160
other white mice were injected with a harmless sugar solution .In two weeks time 39% of
the white mice, who were injected with Serum D contracted the highly contagious and
often fatal disease, jungle fever. Hence, it can be concluded that jungle fever is caused by
some elements similar to the elements in Serum D.

8. The above discussion would be weakened most severely in case it is shown that
A. People contracting jungle fever are usually the victims of the bite of the
South American Lesser Hooded Viper.
B. One among the 160 white mice had already contracted jungle fever prior to
the laboratory experiment.
C. The natural habitats of white mice does not contain any of the elements
found in Serum D.
D. The scientists administered the injections being ignorant of the contents of
the solutions used.
E. The 160 white mice used in the laboratory experiment were kept isolated
from each other.

Ans : B

9. The above argument would be highly empowered in case it were shown that:
A. Some of the elements in Serum D are extracted from the root of a certain
poisonous jungle wildflower.
B. Within a period of two weeks about 40% of the white mice, who were
injected with a harmless sugar solution also contracted jungle fever.
C. Almost all the white mice died within a period of two days after the first
symptoms appeared.
D. Normally the rate of jungle fever among white mice is less than 0.01%.
E. Invariably the blood of the victims of jungle fever victims contains a high
level of a certain toxic substance also found in serum D.

Ans : E

10. Distribution of leaflets and delivering speeches on government property should be


outlawed. Radicals and fanatics have no right to use public property when peddling
their unsavory views.

The argument above is based on the postulate

A. The general public has a special concern in the free exchange of different
political views.
B. Radicals and fanatics prefer the use of public property while propagating
their viewpoint.
C. Every person who hands out leaflets and delivers speeches is a radical or
fanatic.
D. Legal constraints which are applicable to one group need not be equally
applicable to all.
E. Any political activity, which hinders the proper functioning of the
government should not be protected by the law.

Ans : C

Questions 11 - 12

Successfully adjusting to one's environment leads to happiness. War at a universal level


war destroys the weaker people, who are the most unable to adjust to their environment.
Thus, war at the universal level puts weaklings out of their misery and allows more space
for their predators to enjoy life in a better manner. As those actions have to be performed,
which maximize the level of happiness of the greatest number, war at a universal level
should take place.

11. What response would the author of the above discussion come up with, in the case
of the objection that the weaklings far exceed strong people?
I. He would respond with the statement that the person making the objection
is a weakling.
II. He would respond by saying that weaklings will be miserable no matter what
happens.
III. He would respond with the statement that the strong would be frustrated if
the weaklings are destroyed.

A. I only
B. II only
C. III only
D. I and II only
E. II and III only
Ans : E

12. The author's discussion would be greatly if he agreed to which of the following?

I.Technology could change the environment.


II.War at the universal level would be an integral part of the environment.
III.It is possible for the strong to survive without suppressing the weak.

A.I only
B.II only
C.III only
D.I and III only
E.I, II and III only

Ans : A

Analytical Section : Analytical Reasoning


Questions 13 - 15

Come back with us to the real America leaving behind the turmoil of civilization. The real
America is still inhabited by the eagle, the buffalo, the mountain lion and elk; it is still spacious,
sprawling and majestic. Experience the freedom and serenity still to be found in

13. Choose the best option to complete the above statement:


A. the natural beauty of our land
B. the fascinating urban centers
C. the wild terrain of Africa
D. one's own subconscious
E. the great sprawling cities of the Southwest

Ans : A

14. The above paragraph is most likely to appear in which of the following?
A. A Hunter's Guide to The United States
B. Exploring the Great Outdoors
C. The Quiet Beauty of Alaska
D. How the Eagle Became Extinct
E. Returning to America

Ans : D

15. When I am elected, I will work towards effecting those changes for which I have been
fighting all these years. We will work together to do away with the bureaucratic
bogs which have existed ever since my opponent took office. Everyone of you
knows what I stand for; I invite my opponent to ...

For completion of the above statement choose the best option:


A. hand in his resignation graciously
B. make his stance clear
C. stop lying to the public
D. get our city more federal aid
E. extend his support to me

Ans : B

16. We can never make our beliefs regarding the world certain. Even scientific theory
of a most rigorous and well-confirmed nature is likely to change over a decade or
even tomorrow. If we refuse to even try to understand, then it is like resigning from
the human race. Undoubtedly life of an unexamined kind is worth living in other
respects--as it is no mean thing to be a vegetable or an animal. It is also true that a
man wishes to see this speculative domain beyond his next dinner.

From the above passage it is clear that the author believes that

A. men would not do well to speculate


B. progress in the scientific field is impossible
C. one should live life with the dictum 'what will be will be'
D. men should ignore their animal needs
E. men are different from animals as far as their reasoning abilities are
concerned.

Ans : E

Questions 17-21

Two or moreessences out of a stock of five essences-- L, M, N, O, and P are used in making
all perfumes by a manufacturer. He has learned that for a blend of essences to be agreeable
it should comply with all the rules listed below.

A perfume containing L, should also contain the essence N, and the quantity of N should
be twice as that of L.
A perfume containing M, must also have O as one of its components and they should be in
equal proportion.
A single perfume should never contain N as well as O.
O and P should not be used together.
A perfume containing the essence P should contain P in such a proportion that the total
amount of P present should be greater than the total amount of the other essence or
essences used.

17. Among the following which is an agreeable formula for a perfume?


A. One part L, one part P
B. Two parts M, two parts L
C. Three parts N, three parts L
D. Four parts O, four parts M
E. Five parts P, five parts M

Ans : D
18. Adding more amount of essence N will make which of the following perfumes
agreeable?
A. One part L, one part N, five parts P
B. Two parts M, two parts N, two parts P
C. One part M, one part N, one part P
D. Two parts M, one part N, four parts P
E. Two parts N, one part O, three parts P

Ans : A

19. Among the following, the addition of which combination would make an
unagreeable perfume containing two parts N and one part P agreeable?

(A) One part L (B) One part M (C) Two parts N (D) One part O (E) Two
parts P

Ans : E

20. Among the following which combination cannot be used together in an agreeable
perfume containing two or more essences?
A. L and M
B. L and N
C. L and P
D. M and O
E. P and N

Ans : A

21. Among the below mentioned formulas, which can be made agreeable by the
eliminating some or all of one essence ?
A. One part L, one part M, one part N, four parts P
B. One part L, two parts N, one part O, four parts P
C. One part L, one part M, one part O, one part P
D. Two parts L, two parts N, one part O, two parts P
E. Two parts M, one part N, two parts O, three parts P

Ans : B

Analytical Section : Analytical Reasoning


22. Everything that a person does, which is dictated by reason of ignorance is not voluntary.
Involuntary actions are those which produce pain and repentance. Incase a man has
done something in his ignorance and he does not feel vexed due to his action, he
has not acted voluntarily as he was not aware of what he was doing, nor yet
involuntarily since he is not pained.

After reading this passage we can arrive at the conclusion that:


A. A person is not a voluntary agent, if he acts by reason of ignorance and
repents. .
B. If an action is done by reason of ignorance and is not voluntary , then it was
repented.
C. A man is an involuntary agent, if he acts by reason of ignorance.
D. Some actions are either involuntary or not voluntary.
E. If a man is not a voluntary agent, then he acted by reason of ignorance and
repents.

Ans : A

23. Everything that God knows necessarily is, because even what we ourselves know
necessarily is; and, of course, our knowledge is not as certain as God's knowledge
is. But no future contingent thing necessarily is..

Among the following statements, which naturally follows from the above:

1.There are no future contingent things.


2.It is not true that God has knowledge of only necessary things.
3.God has knowledge of no contingent future things.
4.It is not possible for us to know God.
5.God has knowledge of everything. .

Ans : C

Questions 24 - 25

Some lawyers are of the view that the observation of the intrinsic qualities of pornography
in any composition depends on literary criticism and hence it is a matter of opinion. It is rather
odd, though, that in a legal connection, serious critics themselves quite often behave as if
they believed criticism to be a matter of opinion. Why be a critic - and teach in universities
- in case criticism involves nothing but uttering capricious and arbitrary opinions ?

24. In the above argument the author is trying to establish that


A. whether a composition can be called pornographic or not is a matter of
opinion. .
B. it is not a matter of opinion whether a work is pornographic.
C. observance of the qualities of pornography is not dependent on literary
criticism.
D. critics seem hypocritical.
E. critics should not teach at universities.

Ans : D

25. The above discussion would be weakened if it is pointed out that:


A. literary critics are of the opinion that nothing is pornographic.
B. lawyers believe that the observance of the qualities of pornography is a
matter of opinion, as literary critics are not in agreement in this regard.
C. literary critics are not legal authorities.
D. literary critics should not concern themselves with deciding what is
pornographic.
E. literary critics in the teaching profession at the university level are init only for
the money.

Ans : B

Analytical Section : Analytical Reasoning


Questions 26 - 31

Nine individuals - Z, Y, X, W, V, U, T, S and R - are the only candidates, who can serve on
three committees-- A, B and C, and each candidate should serve on exactly one of the
committees.

Committee A should consist of exactly one member more than committee B.


It is possible that there are no members of committee C.
Among Z, Y and X none can serve on committee A.
Among W, V and U none can serve on committee G.
Among T, S and R none can serve on committee C.

26. In case T and Z are the individuals serving on committee B, how many of the nine
individuals should serve on committee C?
A. 3
B. 4
C. 5
D. 6
E. 7

Ans : B

27. Of the nine individuals, the largest number that can serve together on committee C
is
A. 9
B. 8
C. 7
D. 6
E. 5

Ans : D

28. In case R is the only individual serving on committee B, which among the
following should serve on committee A?
A. W and S
B. V and U
C. V and T
D. U and S
E. T and S

Ans : E

29. In case any of the nine individuals serves on committee C, which among the
following should be the candidate to serve on committee A?
A. Z
B. Y
C. W
D. T
E. S

Ans : C

30. In case T, S and X are the only individuals serving on committee B, the total
membership of committee C should be:
A. Z and Y
B. Z and W
C. Y and V
D. Y and U
E. X and V

Ans : A

31. Among the following combinations which could constitute the membership of
committee C?
A. Y and T
B. X and U
C. Y, X and W
D. W, V and U
E. Z, X, U and R

Ans : B

Analytical Section : Logical Reasoning


Directions : Each CAT sample logical reasoning question in this part of the assessment
starts with a reading passage containing the information to be used to choose between correct
and incorrect logical conclusions. These conclusions are based on the information in the
passage. After this reading passage, you are given a lead-in phrase that tells you to choose
from among five different responses. These possible responses are generated by correctly
or incorrectly applying logical thought to the information in the passage at the beginning of
the question. They can be thought of as different ways of completing a sentence that begins
with the lead-in phrase.

Each reading passage is based on actual Bureau of Labor Statistics documents but is not
necessarily a completely accurate representation of BLS work. It is important that you
accept every fact in the reading passage as true, when you evaluate the response choices
offered. You should use only the information in the passage as the basis for accepting or
rejecting any response choices. Be careful not to allow any "facts" that are not clearly
stated in the reading passage, or any outside knowledge you may have of the "facts", to
influence your thinking.

Following are some CAT sample logical reasoning questions.

1. Testifying before the Senate committee investigating charges that cigarette


manufacturers had manipulated nicotine levels in cigarettes in order to addict consumers to
their products, tobacco executives argued that cigarette smoking is not addictive. The
primary reason they gave in support of this claim was that cigarette smoking was
not regulated by the Federal Drug Administration.

For the tobacco executives' argument to be logically correct, which of the following
must be assumed?

A. Substances that are not addictive are not regulated by ...........the Federal
Drug Administration.
B. The tobacco executives lied when they claimed that ...........cigarette
smoking was not addictive.
C. Some addictive substances are not regulated by the ...........Federal Drug
Administration.
D. There is no scientific proof that cigarette smoking is ...........addictive.
E. Substances that are not regulated by the Federal Drug ...........Administration
are not addictive.

Ans : E

2. People should be held accountable for their own behavior, and if holding people
accountable for their own behavior entails capital punishment, then so be it. However,
no person should be held accountable for behavior over which he or she had no
control.

Which of the following is the most logical conclusion of the argument above?

A. People should not be held accountable for the ...........behavior of other


people.
B. People have control over their own behavior.
C. People cannot control the behavior of other people.
D. Behavior that cannot be controlled should not be ...........punished.
E. People have control over behavior that is subject ...........to capital
punishment.

Ans : B

3. There is clear evidence that the mandated use of safety seats by children under age
four has resulted in fewer child fatalities over the past five years. Compared to the
five-year period prior to the passage of laws requiring the use of safety seats,
fatalities of children under age four have decreased by 30 percent.
Which one of the following, if true, most substantially strengthens the argument
above?

A. The number of serious automobile accidents involving ...........children under age


four has remained steady over the ...........past five years.
B. Automobile accidents involving children have decreased ...........sharply
over the past five years.
C. The use of air bags in automobiles has increased by ...........30 percent over
the past five years.
D. Most fatal automobile accidents involving children under ...........age four
occur in the driveway of their home.
E. The number of teenage drivers has increased by 30 ...........percent over the
past five years.

Ans : A

4. Lycopene, glutathione, and glutamine are powerful antioxidants that neutralize the free
radicals that are produced in the body as a result of routine bodily processes. An
excess of these free radicals in your system causes rapid aging because they
accelerate the rate of cellular damage. Aging is simply the result of this damage.
Thus, to slow down aging it is necessary to supplement your diet with these
antioxidants on a daily basis.

Which of the following, if true, most seriously undermines the author's contention?

A. Most persons aren't concerned with the effects of aging ...........until it is too
late to do anything.
B. Exercise associated with normal daily activities effectively
...........neutralizes and dissipates the free radicals that are ...........produced as
a result of routine bodily processes.
C. The cost of antioxidants is exorbitantly high and well ...........beyond the
budget of most consumers.
D. Only overweight people who do not exercise on a daily ...........basis are
likely to have an excess of free radicals in their ...........systems.
E. Smoking cigarettes is one of the main causes of cellular ...........damage in
humans.

Ans : B

5. Is it wrong for doctors to lie about their patients' illnesses? Aren't doctors just like
any other people we hire to do a job for us? Surely, we would not tolerate not being
told the truth about the condition of our automobile from the mechanic we hired to fix
it, or the condition of our roof from the carpenter we employed to repair it. Just as
these workers would be guilty of violating their good faith contracts with us if they
were to do this, doctors who lie to their patients about their illnesses violate these
contracts as well, and this is clearly wrong.

The conclusion of the argument is best expressed by which of the following?


A. Doctors who lie to their patients about their illnesses ...........violate their
good faith contracts with their patients.
B. Doctors often lie to their patients about their illnesses.
C. Doctors are just hired workers like mechanics and ...........carpenters.
D. It is wrong for doctors to lie about their patients' ...........illnesses.
E. Doctors, like mechanics and carpenters, enter into good ...........faith
contracts with us when we hire them.

Ans : D

6. As any economist knows, healthy people pose less of an economic burden to


society than unhealthy people. Not surprisingly, then, every dollar our state
government spends on prenatal care for undocumented immigrants will save
taxpayers of this state three dollars.

Which of the following, if true, would best explain why the statistics cited above
are not surprising?

A. The state's taxpayers pay for prenatal care of all ...........immigrants.


B. Babies born in this state to undocumented immigrant ...........parents are
entitled to infant care benefits from ...........the state.
C. State benefits for prenatal care serve to promote ...........undocumented
immigration.
D. Babies whose mothers did not receive prenatal care ...........are just as
healthy as other babies.
E. Pregnant women who do not receive prenatal care are ...........more likely to
experience health problems than ...........other pregnant women.

Ans : E

7. Beautiful beaches attract people, no doubt about it. Just look at this city's beautiful
beaches, which are among the most overcrowded beaches in the state.

Which of the following exhibits a pattern of reasoning most similar to the one
exhibited in the argument above?

A. Moose and bear usually appear at the same drinking ...........hole at the same time
of day. Therefore, moose and ...........bear must grow thirsty at about the
same time.
B. Children who are scolded severely tend to misbehave ...........more often
than other children. Hence if a child is ...........not scolded severely that child
is less likely to ...........misbehave.
C. This software program helps increase the work ...........efficiency of its users.
As a result, these users ...........have more free time for other activities.
D. During warm weather my dog suffers from fleas ...........more so than during
cooler weather. Therefore, ...........fleas must thrive in a warm environment.
E. Pesticides are known to cause anemia in some people. ...........However,
most anemic people live in regions where ...........pesticides are not
commonly used.
Ans : D

8. Our school district should not spend its money on the new Verbal Advantage
reading program. After all, our students get all the reading practice they need by
studying history and science.

The argument above depends on which the following assumptions?

A. The Verbal Advantage program would not help the ...........students learn
history and science.
B. Other reading programs are just as effective but ...........less expensive than
the Verbal Advantage program.
C. The Verbal Advantage program involves only reading ...........practice.
D. Teaching students history and science is more ...........important than
teaching them reading skills.
E. The students can already read well enough to ...........study history and
science

Ans : C

9. A study of native born residents in Newland found that two-thirds of the children
developed considerable levels of nearsightedness after starting school, while their
illiterate parents and grandparents, who had no opportunity for formal schooling,
showed no signs of this disability.

If the above statements are true, which of the following conclusions is most
strongly supported by them?

A. Only people who have the opportunity for formal ...........schooling develop
nearsightedness.
B. People who are illiterate do not suffer from ...........nearsightedness.
C. The nearsightedness in the children is caused by the ...........visual stress
required by reading and other class work.
D. Only literate people are nearsighted.
E. One-third of the children are illiterate.

Ans : C

10. Newspaper publishers earn their profits primarily from advertising revenue, and
potential advertisers are more likely to advertise in newspapers with a wide
circulation—a large number of subscribers and other readers—than with other
newspapers. But the circulation of the newspaper that is currently the most
profitable one in this city has steadily declined during the last two years, while the
circulation of one of its competitors has steadily increased.

Any of the following, if true, would help explain the apparent discrepancy between
the two statements above EXCEPT:
A. Advertisers generally switch from the most widely ...........circulated
newspaper to another one only when the ...........other one becomes the most
widely circulated ...........newspaper instead.
B. Advertising rates charged by the most profitable ...........newspaper in the
city are significantly higher than ...........those charged by its competitors.
C. The most profitable newspaper in the city receives ...........revenue from its
subscribers as well from advertisers.
D. The circulation of the most profitable newspaper ...........in the city is still
greater than than of any of its ...........competitors.
E. The number of newspapers competing viably with the ...........most
profitable newspaper in the city has increased ...........during the last two
years.

Ans : E

Analytical Section : Logical Reasoning


11. Although most of the fastest growing jobs in today's economy will require a college
degree, many of the new jobs being created-from home health aide to desktop publisher-
require knowledge other than that gained from earning a degree. For workers in
those jobs, good basic skills in reading, communication, and mathematics play an
important role in getting a job and developing a career.

From the information given above it can be validly concluded that, in today's
economy,

A. skills in reading, communication, and mathematics play an important role in


developing a career as a desktop publisher
B. the majority of the new jobs being created require knowledge other than that
gained from earning a college degree
C. a job as a home health aide will rely more on communication skills than on
basic skills in reading and mathematics
D. if a job is one of the fastest growing jobs, it will require a college degree
E. desktop publisher jobs and home health aide jobs are not among the fastest
growing jobs

Ans : A

12. According to the National Agricultural Aviation Society (NAAS), without the use
of crop protection products to control insects, weeds, and diseases, crop yields per
acre will drop by more than 50 percent. The first aerial application of insecticide
occurred in 1921, and it was a huge success. By contrast, in today's economy all
aircraft that are classified as aerial applicators do more than just apply insecticide;
today, they also spread seed and apply fertilizer.

From the information given above it CANNOT be validly concluded that

A. in today's economy, if an aerial applicator is used, then it will be able to


spread seed and to apply fertilizer
B. according to the NAAS, if crop yields per acre never drop by more than 50
percent, then crop protection products have been used to control insects,
weeds, and diseases
C. in today's economy, any aircraft that cannot be used to apply fertilizer
cannot be classified as an aerial applicator
D. in 1921, if an aircraft was used for the application of insecticide, then it was
not also used to spread seed
E. according to the NAAS, if crop yields per acre drop by more than 50
percent, then crop protection products have not been used to control insects,
weeds, and diseases.

Ans : E

13. No national productivity measures are available for underground industries that
may exist but remain unreported. On the other hand, at least some industries that
are run entirely by self-employed industrialists are included in national productivity
measures.

From the information given above, it can be validly concluded that

A. there are at least some industries run entirely by self-employed industrialists


that are underground industries
B. no industries that are run entirely by self-employed industrialists operate
underground
C. there are at least some industries other than those run entirely by self-
employed industrialists that are underground industries
D. there are at least some industries run entirely by self-employed industrialists
that are not underground industries
E. there are at least some underground industries for which national
productivity measures are available

Ans : D

14. Lou observes that if flight 409 is canceled, then the manager could not possibly
arrive in time for the meeting. But the flight was not canceled. Therefore, Lou
concludes, the manager will certainly be on time. Evelyn replies that even if Lou's
premises are true, his argument is fallacious. And therefore, she adds, the manager
will not arrive on time after all.

Which of the following is the strongest thing that we can properly say about this
discussion?

A. Evelyn is mistaken in thinking Lou's argument to be fallacious, and so her


own conclusion is unwarranted.
B. Evelyn is right about Lou's argument, but nevertheless her own conclusion
is unwarranted.
C. Since Evelyn is right about Lou's argument, her own conclusion is well
supported.
D. Since Evelyn is mistaken about Lou's argument, her own conclusion must
be false.
E. Evelyn is right about Lou's argument, but nevertheless her own conclusion
is false.

Ans : B

15. Sally has never received a violation from the Federal Aviation Administration
during her 16-year flying career. Sally must be a great pilot.

Which of the following can be said about the reasoning above?

A. The definitions of the terms create ambiguity.


B. The argument uses circular reasoning.
C. The argument works by analogy.
D. The argument is built upon hidden assumptions.
E. This is an example of an argument that is directed against the source of the
claim rather than the claim itself.

Ans : D

16. The Japanese economic model created strong domestic industries through subsidies
from its Ministry of Trade and by closing off competitive foreign firms to its
domestic market. This strategy promised to help economic growth by incubating
domestic industries. New Japanese industries could count on a known local demand
and would be protected from competition by tariffs and other barriers. The program
could reduce the amount of imports and therefore improve the nation's balance of
trade.

Which of the following, based on the passage above, is a weakness in this


economic strategy?

A. A protectionist policy will create animosity among other nations.


B. Fast growth of small industries will create a class of millionaires and
increase the inequality of income.
C. Subsidies and import constraints keep domestic prices high and impose a
burden on consumers.
D. Quotas are more regressive than tariffs.
E. The demand for the products made by the incubated industries would not be
known.

Ans : C

17. Historically, famines have generally been followed by periods of rising wages,
because when a labor force is diminished, workers are more valuable in accordance
with the law of supply and demand. The Irish potato famine of the 1840s is an
exception; it resulted in the death or emigration of half of Ireland's population, but
there was no significant rise in the average wages in Ireland in the following
decade.

Which one of the following, if true, would LEAST contribute to an explanation of


the exception to the generalization?
A. Improved medical care reduced the mortality rate among able-bodied adults
in the decade following the famine to below prefamine levels.
B. Eviction policies of the landowners in Ireland were designed to force
emigration of the elderly and infirm, who could not work, and to retain a
high percentage of able-bodied workers.
C. Advances in technology increased the efficiency of industry and agriculture,
and so allowed maintenance of economic output with less demand for labor.
D. The birth rate increased during the decade following the famine, and this
compensated for much of the loss of population that was due to the famine.
E. England, which had political control of Ireland, legislated artificially low
wages to provide English-owned industry and agriculture in Ireland with
cheap labor.

Ans : D

18. Cars are safer than planes. Fifty percent of plane accidents result in death, while
only one percent of car accidents result in death.

Which of the following, if true, would most seriously weaken the argument above?

A. Planes are inspected more often than cars.


B. The number of car accidents is several hundred thousand times higher than the
number of plane accidents.
C. Pilots never fly under the influence of alcohol, while car drivers often do.
D. Plane accidents are usually the fault of air traffic controllers, not pilots.
E. Planes carry more passengers than cars do.

Ans : B

19. The body of anyone infected by virus X will, after a week, produce antibodies to fight
the virus; the antibodies will increase in number for the next year or so. There is
now a test that reliably indicates how many antibodies are present in a person's
body. If positive, this test can be used during the first year of infection to estimate to
within a month how long that person has had the virus.

Which one of the following conclusions is best supported by the statements above?

A. Antibodies increase in number only until they have defeated the virus.
B. Without the test for antibodies, there is no way of establishing whether a
person has virus X.
C. Antibodies are produced only for viral infections that cannot be fought by
any other body defenses.
D. If a person remains infected by virus X indefinitely, there is no limit to the
number of antibodies that can be present in the person's body.
E. Anyone infected by virus X will for a time fail to exhibit infection if tested
by the antibody test.

Ans : E
20. Ever since I arrived at the college last week, I've been shocked by the poor behavior
of the students. The student population is completely lacking in proper social skills.

Which of the following, if true, would weaken the above conclusion?

A. Students who are away from their parents often exhibit rude behavior.
B. The college numbers over 50,000 students.
C. The narrator is a student and has interacted with many students.
D. Social skills should not be expected of college students.
E. The narrator was reluctant to stay at the college.

Ans : B

Analytical Section : Logical Reasoning


21. A study of a math program implemented in several pre-schools indicates that
children who received the specialized Math Plus math education between the ages
three and five had significantly higher math scores in 3rd and 4th grade than their
classmates who did not receive this instruction. The proponents of the math argue
that the introduction of this program for all children age three to five will significantly
improve their chances for success in school.

Which of the following, if true, would most seriously weaken the above argument?

A. Most parents send their children to preschool for social development and do not
have a clear idea about what types of education they want for their children.
B. Cognitive abilities of 3- to 5-year-old children are constantly changing.
C. The children in the pre-schools that were studied had previously been exposed
to another math enrichment program.
D. Children are not really interested in enrichment programs in preschool.
E. The cost factor needs to be specified and established before a large scale
program can be undertaken.

Ans : C

22. The symptoms of mental disorders are behavioral, cognitive, or emotional problems. Some
patients with mental disorders can be effectively treated with psychotherapy, but it
is now known that in some patients' mental disorders result from chemical imbalances
affecting the brain. Thus, these patients can be effectively treated only with
medication that will reduce or correct the imbalance.

The argument depends on assuming which one of the following?

A. Treatment by psychotherapy can produce no effective reduction in or


correction of chemical imbalances that cause mental disorders.
B. Treatment with medication always shows faster results for patients with
mental disorders than does treatment with psychotherapy
C. Most mental disorders are not the result of chemical imbalances affecting
the brain.
D. Medication is always more effective in treating patients with mental
disorders than is psychotherapy.
E. Treatment with psychotherapy has no effect on mental disorders other than a
reduction of the symptoms.

Ans : A

23. Dear Editor: I feel obliged to comment on the unfair review you published last
week written by Robert Duxbury. Your readers should know that Mr. Duxbury
recently published his own book that covered the same topic as my book, which
you asked him to review. It is regrettable that Mr. Duxbury should feel the need to
belittle a competing work in the hope of elevating his own book.

The author of the letter above makes her point by employing which method of
argument?

A. Attacking the motives of the author of the unfavorable review.


B. Attacking the book on the same topic written by the author of the review.
C. Contrasting her own book with that written by the author of the review.
D. Questioning the judgment of the author of the unfavorable review.
E. Stating that her book should not have been reviewed by the author of a
competing work.

Ans : A

24. The government of Zunimagua has refused to schedule free elections, release
political prisoners, or restore freedom of speech; therefore, no more financial aid from
the United States should be provided to Zunimagua.

Which of the following is an assumption made in the argument above?

A. Withdrawal of U.S. aid from Zunimagua will force a change in the policies
of its government.
B. The people of Zunimagua would be better off if their present despotic
government were overthrown.
C. The government of Zunimagua is dependent on continued U.S. aid for its
existence.
D. U.S. aid should be given only to countries willing to adopt policies in line
with U.S. interests and goals.
E. U.S. aid should be withdrawn from any country that refuses to operate its
government along democratic lines.

Ans : E

25. Many people argue that the death penalty deters murder. However, the notorious killer
Ned Grandy deliberately moved to a state that imposes the death penalty just before
embarking on a series of ferocious murders. Thus, it seems clear that the existence
of the death penalty does not serve as a deterrent to murder.
The argument above may best be characterized as:

A. an appeal to emotion.
B. a flawed analogy.
C. a general conclusion based on a specific example.
D. circular reasoning.
E. an application of a general principle to a specific example.

Ans : C

26. Steve and JoAnne are both members of a certain club, though they are not speaking
to each other and refuse to work with each other. Cecily, the club president, is
appointing members to the fundraising committee, but she has resolved that she
will not appoint anyone without his or her explicit consent. Steve tells Cecily, "I
will not consent to appointment on that committee unless I know whether JoAnne is
to be a member of it." And JoAnne says, "I will not consent to be a member of that
committee unless I know whether Steve will be appointed to it."

If all three of these people stick by these resolutions, then:

A. Neither of them can be appointed to the committee.


B. The situation described in the scenario cannot arise, because it is inherently
incoherent.
C. They must either both be appointed or both be left out.
D. The committee may finally have one of them, both of them, or neither of
them as members.
E. Either one of them can be appointed, but not both.

Ans : E

27. Russia's aggressive fishing in the prime fishing grounds of the Northern Pacific has
led to a sharp decline in the populations of many fish and a general increase in the
retail price of fish. This same pattern has occurred with far too many of our scarce
vital natural resources, resulting in high prices for many products. It is likely then,
that fish prices will continue to rise in the near future.

In making the argument above, the author relies on all of the following assumptions
EXCEPT:

A. The scarcity of fish is a determining factor in its price.


B. The decline in the number of fish available will result in higher prices for
fish in stores.
C. There will not be any substantial decrease in other costs involved in the
fishing process that could keep the price of fish from increasing.
D. Fish populations will not recover in the near future.
E. Fishing practices can substantially influence the demand for fish.

Ans : E
28. During the past year, Boz Corporation, a cigarette manufacturer, has engaged in a
"corporate image" advertising campaign. One executive now urges that the
advertising be extended for another year because profits have increased by 29%
over the previous year. Another executive, however, is skeptical. She observes that
the increases are typical for the industry over the past year, although none of their
competitors have used corporate image advertising.

The most accurate way of summarizing the second executive's point would be:

A. She argues that the effect may not really be due to its supposed cause
because there has not been a sufficient lapse of time between the cause and
the effect.
B. She argues that the assignment of a cause for this effect is premature,
because there is as yet no well-established theory of such interactions.
C. She argues that corporate image advertising is unprofitable, since it has
evidently benefited competitors as much as the corporation that paid for it.
D. She knows that effective advertising requires a constant influx of new ideas
and approaches, and she argues that one year of corporate image advertising
is enough for awhile.
E. She argues that the effect may not be due to its alleged cause since the same
effect is found elsewhere without that cause.

Ans : E

29. The senate candidate expressed outrage that few judges have any background in
technology, yet they try to resolve cases involving high tech companies. He stated
that not one federal judge has a degree or any experience in computer technology.

A promising response to this concern, arguing that things are not as bad as they
might seem, could involve which of the following claims?

A. Most of the public policy questions in this area are really about the morality
and the value of scientific and technological developments. They do not
require much technical understanding beyond that of a layperson.
B. Computer scientists, by and large, have little interest in politics and public
policy. It would be difficult to find scientists with the degree of commitment
required for a serious contribution to the judicial system.
C. There is a lack of people who are qualified in both technical and legal areas
of expertise.
D. There is very little opportunity for, and indeed little need for, technical
expertise in the judicial branch. There is therefore almost no way for a
technical specialist to rise through the ranks to a top-level position in
government.
E. The rewards of a life as a judge, in terms of both money and prestige, are
not high enough to attract top-flight technical experts to this area.

Ans : A

30. There has been a sharp increase in the subscription prices of many professional and
scholarly journals in the past seven years. Many publishers ascribe the necessity for
these increases to the easy availability of photocopying facilities, which enable
people simply to copy the articles they want rather than buying the journal.

Which of the following, if it is true, would make this explanation more plausible?

A. The great majority of professional and scholarly journals have a massive


backlog of papers awaiting publication.
B. Over the past five years there has been a substantial decline in the number
of individual subscriptions to professional and scholarly journals, while
library subscriptions have remained fairly stable.
C. In the five years immediately preceding the price surge, there was a
substantial decline in the number of individual subscriptions to professional
and scholarly journals, while library subscriptions remained fairly stable.
D. Many libraries have recently begun cutting back on subscriptions to
professional and scholarly journals.
E. In almost every field, several new professional and scholarly journals have
begun publication in the past few years.

Ans : C

Analytical Section : Logical Reasoning


31. Smoking in bed has long been the main cause of home fires. Despite a significant
decline in cigarette smoking in the last two decades, there has been no comparable
decline in the number of people killed in home fires.

Each one of the following statements, if true over the last two decades, helps to
resolve the apparent discrepancy above EXCEPT:

A. Compared to other types of home fires, home fires caused by smoking in


bed usually cause relatively little damage before they are extinguished.
B. Home fires caused by smoking in bed often break out after the home's
occupants have fallen asleep.
C. Smokers who smoke in bed tend to be heavy smokers who are less likely to
quit smoking than are smokers who do not smoke in bed.
D. An increasing number of people have been killed in home fires that started
in the kitchen.
E. Population densities have increased, with the result that one home fire can
cause more deaths than in previous decades.

Ans : B

32. Mrs. Mason is gifted with psychic powers that enable her to foretell future events. In
the past, Mrs. Mason has predicted such actual events as the election of President
Clinton, the stock market crash of 1987, and the St. Louis Cardinals' 1982 World Series
victory. These are just a few of Mrs. Mason's accurate predictions.

The answer to which of the following questions would be most useful in evaluating
the strength of the argument above?
A. What percentage of Mrs. Mason's predictions has come true?
B. Could the election of President Reagan have been predicted without the help
of psychic powers?
C. What is the actual mechanism by which Mrs. Mason's psychic powers are
supposed to operate?
D. How long before the events in question did Mrs. Mason make her accurate
predictions?
E. Do most scientists accept the idea that the power to predict the future
through psychic means really exists?

Ans : A

33. An ingredient in coffee, known as RTC, has been found to inactivate common cold
viruses in experiments. In previous experiments, researchers found that inactivated
common cold viruses can convert healthy cells into cancer cells. It can be
concluded that the use of coffee can cause cancer.

Which one of the following, if true, most seriously weakens the argument?

A. Several teams of scientists performed the various experiments, and all of the
teams had similar results.
B. The carcinogenic effect of RTC could be neutralized by the other
ingredients found in coffee.
C. When RTC kills common cold viruses it weakens the immune system, and it
might thus diminish the body's ability to fight other viruses, including
viruses linked to cancers.
D. If chemists modify the structure of RTC, RTC can be safely incorporated into
medications to prevent the common cold.
E. To lessen the undesirable side effects of chemotherapy, the use of coffee has been
recommended for cancer patients who are free of the common cold virus.

Ans : B

34. Jack Bygrave is an executive at a major South African diamond company that produces
2% of the world's total annual diamond production. The CFO is anxious to
maximize revenues and increase sales. Bygrave, however, believes that increased
production would only drive down the world price of diamonds and lower
revenues.

Which of the following represents the logical flaw in Bygrave's reasoning?

A. Jack connects the price of unrefined diamonds and the price of jewelry-
quality diamonds.
B. He assumes that production goals are similar to financial goals.
C. He assumes that the supply produced by a single company can significantly
alter the aggregate supply for the market.
D. He assumes that seasonal and long term supply are proportional.
E. He correlates long-term and short-term demand.

Ans : C
35. The crux of creativity resides in the ability to manufacture variations on a theme. If
we look at the history of science, for instance, we see that every idea is built upon a
thousand related ideas. Careful analysis leads us to understand that what we choose
to call a new theme or a new discovery is itself always and without exception some
sort of variation, on a deep level, of previous themes.

If all of the statements in the passage are true, each of the following must also be
true EXCEPT:

A. A lack of ability to manufacture a variation on a previous theme connotes a


lack of creativity
B. No scientific idea is entirely independent of all other ideas.
C. Careful analysis of a specific variation can reveal previous themes of which
it is a variation.
D. All great scientific discoverers have been able to manufacture a variation on
a theme.
E. Some new scientific discoveries do not represent, on a deep level, a
variation on previous themes.

Ans : E

36. Studies of fatal automobile accidents reveal that, in the majority of cases in which one
occupant of an automobile is killed while another survives, it is the passenger, not
the driver, who is killed. It is ironic that the innocent passenger should suffer for the
driver's carelessness, while the driver often suffers only minor injuries or none at
all.

Which of the following is an assumption underlying the reasoning in the passage


above?

A. In most fatal automobile accidents, the driver of a car in which an occupant


is killed is at fault.
B. Drivers of automobiles are rarely killed in auto accidents.
C. Most deaths in fatal automobile accidents are suffered by occupants of cars
rather than by pedestrians.
D. Auto safety experts should increase their efforts to provide protection for
those in the passenger seats of automobiles.
E. Automobile passengers sometimes play a contributing role in causing auto
accidents.

Ans : A

37. The editors of Business Today magazine conducted a poll of its readers regarding the
proposed increase in the rate of income tax paid on profits from the sale of stocks.
More than 60% of the readers opposed the proposed tax. The editors announced
that the majority of Americans opposed any increase in the tax on profits from
stock sales.

Which one of the following statements, if true, would most weaken the editor's
conclusion?
A. Some readers of Business Today magazine are citizens of countries other
than the United States.
B. Decisions concerning the income tax laws are made by the Congress rather
than directly by the people.
C. Most of those who earn profits from stock sales are wealthy and can afford
to pay higher taxes.
D. The viewpoints of the vast majority of the readers of Business Today
magazine differ from the views of most Americans.
E. Not all readers of Business Today magazine responded to the editors' poll.

Ans : D

38. Several movie versions of Charles Dickens' Tale of Two Cities have been made. The
original movie version made in 1939 is the best because it is closest in spirit to the
original novel.

An underlying assumption of the argument above is that a movie based on a novel


should:

A. reflect the director's original interpretation of the main themes of the novel.
B. accurately depict the time and place in which the novel is set.
C. feature actors and actresses who closely resemble the characters in the novel
both in body and spirit.
D. faithfully render the details of the plot from the narrator's point of view.
E. capture the true meaning and intention of the novel.

Ans : E

Following are some CAT sample general awareness questions.

1. Which company's famously advertised vision statement is 'The Network is the


Computer'?
A. Cisco Systems
B. Lucent Technologies
C. Sun Microsystems
D. Nortel Networks

Answer : C

2. The software company I-flex Solutions was originally a division of which famous
financial services company?
A. Citicorp
B. ICICI
C. HSBC
D. ABN Amro Bank

Answer : A
3. Which former advertising personality has recently been named the Undersecretary
of State for 'public diplomacy and public affairs' of the US in an exercise to rebrand
the US following the September 11 terrorist strikes?
A. Charlotte Beers
B. Jay Chiat
C. Martin Sorrell
D. George Lois

Answer : A

4. Which premium international range of luggage was recently launched in India by


BlowPlast?
A. Samsonite
B. Strolley
C. American Tourister
D. Delsey

Answer : D

5. This year saw the launch of Yahoo!, Amazon.com and the famous launch of the
Orange mobile phone service in the UK. Which year was this?
A. 1992
B. 1994
C. 1995
D. 1993

Answer : B

6. Henry Ford revolutionised the car market with the first mass- produced car, the
Ford Model T. In which year was it launched?
A. 1924
B. 1912
C. 1908
D. 1897

Answer : C

7. Which company owns the beer brands Haywards 2000, Hi-Five and Lal Toofan?
A. United Breweries
B. Millennium Alcobev
C. Shaw Wallace
D. Mohan Meakins

Answer : C

8. Which company owns the footwear brand - Stryde?


A. Woodland
B. Bata
C. Tata International
D. Hindustan Lever
Answer : C

9. It was the brand that made David Ogilvy famous. Which shirt brand's ads had the
famous man with an eye patch in the 1950s, which catapulted David Ogilvy to
fame?
A. Van Heusen
B. Arrow Shirts
C. Hathaway
D. Dockers

Answer : C

10. Which TV channel has been in the news for its coverage of the Bin Laden crisis,
especially for having carried all broadcasts of Osama Bin Laden to audiences in
West Asia?
A. Star Asia
B. BBC Asia
C. Al Jazeera
D. Khaleed Times

Answer : C

General Awareness
11. In the case of a test tube baby
A. Fertilization takes place inside the test tube
B. Development of the baby takes place inside the test tube
C. Fertilization takes place outside the mother's body
D. Unfertilized egg develops inside the test tube

Answer : C

12. Who elects the President of India?


A. Lok Sabha
B. Rajya Sabha
C. People of India
D. Parliament and State Assemblies

Answer : D

13. The Fundamental Duties were incorporated in the Constitution of India by the
A. First Amendment
B. Tenth Amendment
C. Thirty-second Amendment
D. Forty-second Amendment

Answer : D
14. The speed of light will be minimum while passing through
A. glass
B. vacuum
C. air
D. water

Answer : A

15. The time taken by the Sun to revolve around the center of our galaxy is
A. 50 mn years
B. 100 mn years
C. 250 mn years
D. 365 mn years

Answer : C

16. The oldest monarchy in the world is that of


A. Nepal
B. U K
C. Spain
D. Japan

Answer : D

17. The Vice-President is elected by an electoral college consisting of


A. only elected members of both Housed of Parliament
B. only Rajya Sabha members
C. all members of Parliament and State Legislative Assemblies
D. all members of Lok Sabha and Rajya Sabha

Answer : D

18. In which one of the following states of India is it legal for a Hindu male and illegal
for a Muslim male to have more than one living wife?
A. Nagaland
B. Goa
C. Himachal Pradesh
D. Arunachal Pradesh

Answer : B

19. Where did Buddha die?


A. Lumbini
B. Kusinagara
C. Pavapuri
D. Magadha

Answer : B
20. What was the main difference between the Indus Valley Civilization and Vedic
Civilization?
A. Indus Valley Civilization was urban, while the Vedic Civilization was rural.
B. "Pipal" tree was worshiped in Indus Valley Civilization, while "Burgad" tree
was worshiped in Vedic Civilization.
C. The main emphasis in Indus Valley Civilization was on trade while in the
Vedic Age was on religion.
D. Indus Valley Civilization believed in non-violence while Vedic Civilization
had no hard and fast rules about violence.

Answer : A

General Awareness
21. All of the following are constituents of RNA molecule except
A. Adenine
B. Guanine
C. Thymine
D. Uracil

Answer : C

22. The father of modern biology is


A. Aristotle
B. Darwin
C. Mendel
D. Vesalius

Answer : A

23. Which of the following seeds will normalize blood sugar level?
A. Coriander
B. Mustard
C. Cumin
D. Fenugreek

Answer : D

24. The point at which solid, liquid and gaseous forms of a substance co-exist is called
A. sublimation point
B. distillation point
C. triple point
D. melting point

Answer : C

25. Which among the following substances is used as a lubricant?


A. Quartz
B. Silica
C. Graphite
D. Nickel

Answer : C

26. The weight of an object will be minimum when it is placed at


A. The North Pole
B. The South Pole
C. The Equator
D. The center of the Earth

Answer : D

27. Earth quake waves travel fastest in


A. Soil
B. Molten rock
C. Water
D. Flexible rock

Answer : C

28. The angle between the geographical meridian and magnetic meridian is called
A. Angle of dip
B. Angle of declination
C. Angle of inclination
D. None of the above

Answer : B

29. What is the currency of Mexico?


A. Lira
B. Peso
C. Mexican dollars
D. Krones

Answer : B

30. The Essar group of companies has been promoted by


A. Ruias
B. Ambanis
C. Goenkas
D. Kanorias

Answer : A

General Awareness
31. One of the following liquor brands is not owned by United Breweries
A. Kalyani Black Label
B. Blue Riband
C. McDowells
D. Bagpiper

Answer : D

32. Amtrex air conditioners has a technical collaboration with


A. Sanyo
B. Mitsubishi
C. Hitachi
D. Carrier

Answer : C

33. Marlboro Cigarette is owned by


A. ITC
B. Godfrey-Philips
C. Philip Morris
D. British American Tobacco

Answer : C

34. Who is the CEO of Microsoft?


A. Bill Gates
B. Paul Allen
C. Larry Ellison
D. Steve Ballnes

Answer : D

35. One of the following is not an Insurance Company


A. ICICI Prudential
B. HDFC Natwest
C. OM Kotak Mahindra
D. Birla Sun Life

Answer : B

36. What is the currency of Portugal?


A. Escudo
B. Guilder
C. Mark
D. Schilling

Answer : A

37. One of the following personalities is not associated with the Indian auto sector
A. Suresh Krishna
B. Venu Srinivasan
C. Anand Mahindra
D. Venugopal Dhoot

Answer : D

38. One of the following companies is not in the area of air-conditioning &
refrigeration
A. BPL
B. Thermax
C. Lloyd
D. Carrier Aircon

Answer : C

39. How many countries are a part of the European union?


A. 18
B. 15
C. 21
D. 12

Answer : B

40. Who is the attorney general of United States of America?


A. Ashcroft
B. Powell
C. Rumsfeld
D. Cheney

Answer : A

General Awareness
41. When was the generic domain name (Top Level Domain - TLD) .com introduced?
A. 1989
B. 1985
C. 1994
D. 1991

Answer : B

42. Who is the prime minister of Israel?


A. Benajamin Netanyahu
B. Ariel Sharon
C. Shaul Mofaz
D. Goldamyer

Answer : B
43. What was the significant about the purchase of a kilo of lychees on the French
island of Reunion, located in the Indian Ocean?
A. They were the first lot of lychees exported from India.
B. It was the first official purchase using the new currency Euro.
C. It was done by to commemorate the release of the new France with Princess
Diana's image on it.
D. None of these

Answer : B

44. Which brand had the highest number of Web searches in 2001?
A. Google
B. Marlboro
C. Intel
D. Play Station

Answer : D

45. What is common to all of the following names - Eduardo Camano, Adolfo
Rodriguez Saa, Ramon Puerta and Fernando de la Rua?
A. They were all former Presidents of Argentina.
B. They are the dreaded drug lords of South America
C. They own together 70% of the world's Silver mines.
D. None of these

Answer : A

46. The video games X box is a product of


A. Sega
B. Sony
C. Intel
D. Microsoft

Answer : D

47. It was acknowledged as the second-most dangerous computer virus in history, after
the Love Bug virus. Name this virus, which struck in 2001
A. Melissa
B. Code Red
C. C-Brain
D. Major Domo

Answer : B

48. What does the letters XP stand for in the product Microsoft XP?
A. Extended product
B. Extra Pampering
C. Experience
D. Entry level product
Answer : C

49. Which business and media tycoon won the elections to become Head of State in
Italy amidst widespread clouds of scandal?
A. Guillani Giovanni
B. Joe Pacci
C. Antonio Machiaveli
D. Silvio Berusconi

Answer : D

50. Which city is hosting the 14th Asian Games in 2004?


A. Manila
B. Busan
C. Beijing
D. Bangkok

Answer : B

Quantitative Section : Data Interpretation


Questions 1 - 5 refers to the following table:

PROFILE OF CONGRESS IN YEAR X


(total membership: 535)

House of
Senate
Representatives
Party
292 Democratic 62
143 Republican 38
435 TOTAL 100
Sex
418 Male 100
17 Female 0
Age
27 Youngest 34
77 Oldest 80
Average
48 54
(arithmetic mean)
Religion
255 Protestant 69
107 Catholic 12
18 Jewish 5
4 Mormon 3
51 Other 11
House of Senate
Representatives
Profession
215 Lawyer 63
Business Executive
81 or Banker 15
45 Educator 6
14 Farmer or Rancher 6
22 Career Government 0
24 Official 4
Journalist or
2 0
Communications
1 Executive 1
0 Physician 2
Veterinarian
6 0
Geologist
Worker or Skilled
25 Tradesperson 3
Other
Ethnic Group
17 Black American 1
2 Asian American 3
4 Hispanic American 0

1. In the Senate, if 25 male members were replaced by 25 female members, the ratio
of male members to female members would be
A. 4 to 1
B. 3 to 1
C. 3 to 2
D. 2 to 1
E. 1 to 1

Ans : B

2. Approximately what percent of the members of Congress are lawyers?


A. 63%
B. 58%
C. 56%
D. 52%
E. 49%

Ans : D

3. If 5 senators are Catholic Democrats, how many senators are neither Catholic nor
Democratic?
A. 79
B. 74
C. 69
D. 31
E. 21
Ans : D

4. If all lawyers and all women in the House of Representatives vote for the passage of a
bill, how many more votes will be needed for a majority?
A. 435
B. 220
C. 3
D. 0
E. It cannot be determined from the information given.

Ans : E

5. Which of the following can be inferred from the information given in the chart?

I.More than 80 percent of the men in Congress are members of the House of
Representatives.
II.The percent of members who are categorized as farmers or ranchers is greater
for the House of Representatives than for the Senate.
III.The median age in the Senate is 57.

A.I only
B.II only
C.III only
D.I and II
E.I and III

Ans : A

Quantitative Section : Data Interpretation


Questions 6 - 7 refers to the following table:

PERCENT CHANGE IN DOLLAR AMOUNT OF SALES


IN CERTAIN RETAIL STORES FROM 1977 TO 1979

Percent Change

From 1977 From 1978


Store
to 1978 to 1979
P +10 -10
Q -20 +9
R +5 +12
S -7 -15
T +17 -8

6. In 1979, for which of the stores was the dollar amount of sales greater than that of
any of the others shown?
A. P
B. Q
C. R
D. S
E. It cannot be determined from the information given.

Ans : E

7. In store T, the dollar amount of sales for 1978 was approximately what percent of
the dollar amount of sales for 1979?
A. 86%
B. 92%
C. 109%
D. 117%
E. 122%

Ans : C

Questions 8 - 9 refers to the following Figure:

8. Of every dollar received by the federal government, how much (in cents) is from
coporate sources?
A. 32
B. 70
C. 30
D. 35
E. 29

Answer : 1

9. what percentage of the federal revenue is derived from borrowings?


A. 0.2%
B. 0.02%
C. 2.7%
D. 1.2%
E. 2.5%
Answer : 3

Quantitative Section : Data Interpretation


Questions 10 - 11 refers to the following table:

DIRECTIONS: The following question are based on the bellow table, which shows per
capita Mean Expenditure, Per capita Food expenditure, Number of Households and Per
capita cereal consumption, in both quantity and value, for different expenditure classes of
rural India. The sampled 41597 households are divided into 12 expenditure classes, starting
from less than Rs.65 per month per capita and ending at more than Rs.385 per capita per
month.

10. According to the results of this sample survey, what is the proportion of total
expenditure on food to total expenditure for all the sampled households taken
together?
A. 58%
B. 36.7%
C. 63.3%
D. 71%
E. Cannot be determined

Answer : 3

11. What is the difference, approximately, between the gross expenditure of the
sampled households in the Rs.95-110 expenditure class and in the Rs.180-215
expenditure class?
A. 372000
B. 448000
C. 496000
D. 93.8
E. 52.3

Answer : A

Questions 12 - 13 refers to the following Graph:


GRAPH SHOWS EXPENDITURE ON ARMS BY DIFFERENT COUNTRIES (VALUE IN
DOLLARS '000 MILLIONS)

12. The amount spent by country C in 1983 is what percentage more than the amount
spent by Countries A and B together in 1977? (Find approximately)
A. 50%
B. 179%
C. 75%
D. 13%
E. 70%

Answer : C

13. Which of the following statements must be true?

i.Country A spends minimum amount of its budget on arms.


ii.Throughout, Country C has spent the maximum amount on arms during the
years shown.
iii.An examination of the information for the last 3 years reveals that generally
all 3 countries are reducing their expenditure on arms.

A.i only.
B.i and ii only
C.i and iii only
D.ii and iii only
E.None of the statements above.

Answer : E

You might also like